You are on page 1of 167

See discussions, stats, and author profiles for this publication at: https://www.researchgate.

net/publication/304432282

Financial Derivatives

Book · December 2016

CITATIONS READS

0 8,491

1 author:

Buerhan Saiti
Istanbul Sabahattin Zaim University
184 PUBLICATIONS   475 CITATIONS   

SEE PROFILE

Some of the authors of this publication are also working on these related projects:

econometric testing of issues in finance and economics View project

Islamic Crowdfunding: Fundamentals, Developments, and Challenges View project

All content following this page was uploaded by Buerhan Saiti on 15 August 2017.

The user has requested enhancement of the downloaded file.


BBFD4103
FINANCIAL
DERIVATIVES
Dr Buerhan Saiti
Project Directors: Prof Dato’ Dr Mansor Fadzil
Prof Dr Wardah Mohamad
Open University Malaysia

Module Writer: Dr Buerhan Saiti


International Islamic University Malaysia

Moderator: Ratna Khuzaimah Mohamad


Open University Malaysia

Developed by: Centre for Instructional Design and Technology


Open University Malaysia

First Edition, December 2016


Copyright © Open University Malaysia (OUM), December 2016, BBFD4103
All rights reserved. No part of this work may be reproduced in any form or by any means
without the written permission of the President, Open University Malaysia (OUM).
Table of Contents
Course Guide ix-xiii

Topic 1 Introduction to Derivatives Markets 1


1.1 Common Derivatives Instruments 2
1.2 Evolution of Derivatives 3
1.2.1 Advent of Swaps 5
1.3 Exchange Traded Derivatives 6
1.4 Over-the-Counter (OTC) Derivatives 6
1.5 Exchange Traded Derivatives versus 7
Over-the-Counter Derivatives
1.6 Main Players in Derivatives Market 8
Summary 9
Key Terms 10
Reference 10

Topic 2 Mechanics of Futures Markets 11


2.1 Specification of Futures Contract 12
2.2 Convergence of Futures Price to Spot Price 13
2.3 Forward and Futures Contracts 14
2.3.1 Futures Contracts 15
2.4 Mechanics of Futures and Operation of Margins 16
2.4.1 Mechanics of Futures 17
2.4.2 Margining and Marking-to-Market 19
2.5 Newspaper Quotes 22
Summary 24
Key Terms 25
References 25

Topic 3 Determination of Forward and Futures Prices 26


3.1 Investment Assets versus Consumption Assets 27
3.2 Short Selling 27
3.3 Measuring Interest Rates 29
3.4 Forward and Futures Prices for Investment Assets 29
3.4.1 Forward Price for Investment Assets 29
3.4.2 Futures Prices for Investment Assets 31
3.5 Valuing Forward Contracts 32
3.6 Are Forward Prices and Futures Prices Equal? 33
iv  TABLE OF CONTENTS

Summary 34
Key Terms 35
References 35

Topic 4 Hedging Strategies Using Futures 36


4.1 Basic Principles of Hedging 37
4.2 Arguments For and Against Hedging 38
4.3 Basis Risk 39
4.4 Minimum Variance Hedge Ratio 40
4.5 Stock Index Futures (SIF) Contract 43
4.5.1 Why Use Stock Index Futures (SIF) Contracts? 43
4.5.2 The Main Players in Stock Index Futures (SIF) 45
Markets
4.5.3 Pricing of Stock Index Futures (SIF) Contracts 45
4.5.4 Application of Stock Index Futures (SIF) Contracts 47
4.6 Single Stock Futures (SSF) 50
4.6.1 Contract Specifications of Single Stock Futures (SSF) 51
Summary 54
Key Terms 55
References 55

Topic 5 Swaps 56
5.1 Interest Rate Swaps (IRS) 56
5.1.1 Why Use Interest Rate Swaps (IRS)? 59
5.2 Applications of Interest Rate Swaps (IRS) 60
5.2.1 Arbitrage Using IRS 60
5.2.2 Hedging Using IRS 62
5.3 Valuation of Interest Rate Swaps (IRS) 64
5.4 Currency Swaps 65
5.5 Risks of Interest Rate and Currency Swaps 67
Summary 68
Key Terms 69
References 69

Topic 6 Mechanics of Options Markets 70


6.1 Options 71
6.2 Specifications of Stock Options 72
6.3 Call Options 74
6.3.1 Long Call 75
6.3.2 Short Call 75
6.4 Put Options 76
6.4.1 Long Put 77
6.4.2 Short Put 77
TABLE OF CONTENTS  v

6.5 Option Moneyness 79


6.6 Trading, Commissions and Margins 80
6.7 The Options Clearing Corporation (OCC) 81
Summary 81
Key Terms 82
References 82

Topic 7 Trading Strategies Involving Options 83


7.1 Uncovered or Naked Positions 84
7.2 Hedge Strategies 85
7.2.1 Portfolio Insurance ă Long Stock Position 85
7.2.2 Hedging Exposed to a Short Stock Position 87
7.3 Spread Strategies 89
7.3.1 Bull Call Spread 90
7.3.2 Bull Put Spread 92
7.3.3 Summary of Bull Spreads 93
7.3.4 Bear Call Spread 94
7.3.5 Bear Put Spread 95
7.3.6 Summary of Bear Spreads 96
7.4 Combination Strategies 98
7.4.1 Straddle Strategy 98
7.4.2 Strangle Strategy 102
7.5 Strategies by Market Outlook 107
Summary 108
Key Terms 109
References 109

Topic 8 Option Pricing Models 110


8.1 Binomial Option Pricing Model (BOPM) ă Call Option 111
8.1.1 Two-step Binomial Trees 112
8.1.2 Three-step Binominal Trees 113
8.2 Probabilities and Volatility 114
8.2.1 Probability Changes 114
8.2.2 Volatility Changes 115
8.3 Binomial Option Pricing Model (BOPM) ă Put Option 116
8.4 The Black-Scholes Option Pricing Model (BSOPM) 117
8.4.1 Underlying Assumptions of the Black-Scholes 117
Option Pricing Model (BSOPM)
8.4.2 The Black-Scholes Option Pricing Model (BSOPM) 118
for a Call Option
8.4.3 The Black-Scholes Option Pricing Model (BSOPM) 120
for a Put Option
8.5 Determinants of Option Prices 121
vi  TABLE OF CONTENTS

8.6 Implied Volatilities 123


Summary 123
Key Terms 124
References 124

Topic 9 Options on Stock Indices, Currencies and Futures 125


9.1 Stock for Paying a Known Dividend Yield 126
9.2 Stock Index Options 126
9.3 Using Stock Index Options for Portfolio Insurance 128
9.4 Currency Options 130
9.4.1 Valuing European Currency Options 131
9.5 Futures Options 132
9.5.1 Valuing European Futures Options 133
Summary 134
Key Terms 134
References 135

Topic 10 Derivative Instruments and Islamic Finance 136


10.1 The Prerequisites of Islamic Financial Instruments 137
10.2 Islamic Financial Instruments Similar to 138
Derivative Instruments
10.2.1 BayÊ al-Salam 138
10.2.2 BayÊ al-IstisnaÊ and JuÊalah 140
10.2.3 Istijrar Contract 141
10.2.4 Bai-al-Urbun 144
10.3 Pricing of a Sukuk Ijara with Warrants/Embedded Options 145
10.4 Current Derivative Instruments from Islamic Perspective 147
10.4.1 Opinions on Futures Contracts 148
10.4.2 Opinions on Option Contracts 149
Summary 150
Key Terms 151
References 151
COURSE GUIDE
COURSE GUIDE  ix

COURSE GUIDE DESCRIPTION


You must read this Course Guide carefully from the beginning to the end. It tells
you briefly what the course is about and how you can work your way through
the course material. It also suggests the amount of time you are likely to spend in
order to complete the course successfully. Please keep on referring to Course
Guide as you go through the course material as it will help you to clarify
important study components or points that you might miss or overlook.

INTRODUCTION
BBFD4103 Financial Derivatives is one of the courses offered by OUM Business
School at Open University Malaysia (OUM). This course is worth 3 credit hours
and should be covered over 8 to 15 weeks.

COURSE AUDIENCE
This course is offered to all learners taking the Bachelor of Banking and Finance.
This module aims to impart the fundamentals of financial derivatives and covers
the important topics such as derivatives markets, forms of derivatives, mechanics
of futures market, determination of forward and futures prices, hedging
strategies using futures, swaps, mechanics of options markets, factors affecting
option value and option pricing models and as well as the sharia-compliant
derivatives.

As an open and distance learner, you should be acquainted with learning


independently and being able to optimise the learning modes and environment
available to you. Before you begin this course, please confirm the course material,
the course requirements and how the course is conducted.

STUDY SCHEDULE
It is a standard OUM practice that learners accumulate 40 study hours for every
credit hour. As such, for a three-credit hour course, you are expected to spend
120 study hours. Table 1 gives an estimation of how the 120 study hours could be
accumulated.
x  COURSE GUIDE

Table 1: Estimation of Time Accumulation of Study Hours

Study
Study Activities
Hours
Briefly go through the course content and participate in initial discussions 5
Study the module 60
Online participation 20
Revision 15
Assignment(s) and Examination(s) 20
TOTAL STUDY HOURS ACCUMULATED 120

COURSE OUTCOMES
By the end of this course, you should be able to:

1. Differentiate the various types of derivatives;

2. Recognise the mechanics of derivatives trading;

3. Support the importance of hedging and strategies for risk management;


and

4. Summarise the usefulness of option pricing models.

COURSE SYNOPSIS
This course is divided into 10 topics. The synopsis for each topic is listed as
follows:

Topic 1 starts off with the differences between exchange traded and over-the-
counter markets. Then, you will be introduced to forward, futures and other
derivatives. More importantly, you will be introduced to the issues associated
with forward contracts. Last but not the least, you will get to know the necessity
of futures contracts over forward contracts.

Topic 2 discusses the specifications of futures contract and the operation


mechanisms of futures contract. By the end of this topic, you should be able to
differentiate between forward and futures contracts with appropriate examples.
COURSE GUIDE  xi

Topic 3 serves as a fundamental topic for forward and futures prices. Hopefully,
you will be able to differentiate between investment and consumption assets and
able to define short selling and familiarise with how to determine and value the
forward prices.

Topic 4 introduces the basic principles of hedging and the arguments for and
against hedging. More importantly, by the end of this topic, you will be able to
analyse basis risk and minimum variance hedge ratio and discuss the stock index
futures.

Topic 5 explores the basic principles of swap and explains the comparative
advantage arguments. Furthermore, you can value the interest rate and currency
swaps and discuss the credit risk by completion of this topic.

Topic 6 elaborates on the specification of stock options and explains the trading
mechanisms and commissions and margins. By the end of this topic, you will be
able to explain and differentiate between call and put options and describe the
option moneyness.

Topic 7 serves as an application of option contracts. Therefore, you should able to


apply the option knowledge on option strategies and valuate a single option and
a stock strategy. Last but the least, you should comprehend the option spreads
strategies and appreciate the flexibility of options.

Topic 8 explores several ways to price or value the options. By completion of this
topic, you should able to calculate the option prices by suing one and two steps
binomial model and interpret and calculate the option prices by applying the
Black-Scholes pricing model. Also, you should able to comprehend on the
American options.

Topic 9 serves as another application topic. Thus, you should be able to apply
options on stock indices, currencies and futures as well as examine the option
pricing models for stock indices, currencies and futures. At the same time, you
should illustrate the application of stock index options on portfolio insurance.

Topic 10 introduces some forms of sharia-compliant derivatives so that you will


recognise the issue and challenges in sharia-compliant derivatives. Last but not
the least, you should have a good understanding of several sharia-compliant
structured products and their mechanisms to be used in risk management.
xii  COURSE GUIDE

TEXT ARRANGEMENT GUIDE


Before you go through this module, it is important that you note the text
arrangement. Understanding the text arrangement will help you to organise your
study of this course in a more objective and effective way. Generally, the text
arrangement for each topic is as follows:

Learning Outcomes: This section refers to what you should achieve after you
have completely covered a topic. As you go through each topic, you should
frequently refer to these learning outcomes. By doing this, you can continuously
gauge your understanding of the topic.

Self-Check: This component of the module is inserted at strategic locations


throughout the module. It may be inserted after one sub-section or a few sub-
sections. It usually comes in the form of a question. When you come across this
component, try to reflect on what you have already learnt thus far. By attempting
to answer the question, you should be able to gauge how well you have
understood the sub-section(s). Most of the time, the answers to the questions can
be found directly from the module itself.

Activity: Like Self-Check, the Activity component is also placed at various


locations or junctures throughout the module. This component may require you to
solve questions, explore short case studies, or conduct an observation or research.
It may even require you to evaluate a given scenario. When you come across an
Activity, you should try to reflect on what you have gathered from the module and
apply it to real situations. You should, at the same time, engage yourself in higher
order thinking where you might be required to analyse, synthesise and evaluate
instead of only having to recall and define.

Summary: You will find this component at the end of each topic. This component
helps you to recap the whole topic. By going through the summary, you should
be able to gauge your knowledge retention level. Should you find points in the
summary that you do not fully understand, it would be a good idea for you to
revisit the details in the module.

Key Terms: This component can be found at the end of each topic. You should go
through this component to remind yourself of important terms or jargon used
throughout the module. Should you find terms here that you are not able to
explain, you should look for the terms in the module.

References: The References section is where a list of relevant and useful


textbooks, journals, articles, electronic contents or sources can be found. The list
can appear in a few locations such as in the Course Guide (at the References
section), at the end of every topic or at the back of the module. You are
COURSE GUIDE  xiii

encouraged to read or refer to the suggested sources to obtain the additional


information needed and to enhance your overall understanding of the course.

PRIOR KNOWLEDGE
There is no prior knowledge needed.

ASSESSMENT METHOD
Please refer to myINSPIRE.

REFERENCES
Bacha, O. I. (2012). Financial derivatives: Markets and applications in Malaysia
(3rd ed). Shah Alam, Malaysia: McGraw-Hill.

Bacha, O. I., & Mirakhor, A. (2013). Islamic capital markets: A comparative


approach. Singapore: John Wiley & Sons.
Hull, J. C. (2012). Options, futures, and other derivatives (8th ed.). Boston, MA:
Prentice Hall.

TAN SRI DR ABDULLAH SANUSI (TSDAS) DIGITAL


LIBRARY
The TSDAS Digital Library has a wide range of print and online resources for the
use of its learners. This comprehensive digital library, which is accessible
through the OUM portal, provides access to more than 30 online databases
comprising e-journals, e-theses, e-books and more. Examples of databases
available are EBSCOhost, ProQuest, SpringerLink, Books247, InfoSci Books,
Emerald Management Plus and Ebrary Electronic Books. As an OUM learner,
you are encouraged to make full use of the resources available through this
library.
xiv  COURSE GUIDE
T op i c  Introduction to
1 Derivatives
Markets
LEARNING OUTCOMES
By the end of the topic, you should be able to:
1. Identify the four common derivative instruments;
2. Describe the evolution of derivatives;
3. Differentiate between exchange traded derivatives and over-the-
counter derivatives; and
4. Identify the main players in the derivatives market.

 INTRODUCTION
Welcome to BBFD4103 Financial Derivatives. Before we go further, what do
financial derivative instruments mean? Financial derivative instruments are
basically financial instruments, where we derive their values from the value of an
underlying asset. A derivative instrument in itself holds little value and its entire
value is dependent on the underlying asset (Bacha, 2012).

Let us say, I purchase and hold a crude oil futures contract. The value of this
contract will rise and fall as the value or price of spot crude oil rises or falls. In
this case, the underlying asset is crude oil, and the value of the crude oil futures
contract that I am holding will increase or decrease in value depending on the
spot value of the underlying asset.

What else do you need to know about derivatives markets? Well, you will be
introduced to common derivative instruments, the evolution of derivatives as
well as exchange traded derivatives and over-the-counter derivatives. This is
2  TOPIC 1 INTRODUCTION TO DERIVATIVES MARKETS

followed by the comparison between exchanged traded derivatives and over-the-


counter derivatives and the main players in the derivatives market. Are you
ready to discover more on the world of derivatives markets? I hope so. Let us
continue the lesson.

1.1 COMMON DERIVATIVES INSTRUMENTS


Firstly, let us get to know the common derivatives instruments. There are four
that you need to be familiarised with as listed in Figure 1.1.

Figure 1.1: Four common derivatives instruments

These four instruments are further explained as follows:

(a) Forward Contract


In the forward contract, two parties negotiate to undertake a transaction at
a future date but the price is determined at the present day.

(b) Futures Contract


If we look from the operational perspective, the futures contract is simply a
standardised and exchange traded form of forward contract.

Similar to the forward contract, the futures contract is also an agreement


between two parties to undertake a transaction at a future date but the
price is determined at the time of contract initiation. The difference is that
futures are standardised and are exchange traded contracts.

(c) Option Contract


An option contract enables the holder (buyer or seller) the right but not the
obligation to buy or sell the underlying asset at a predetermined price.
There are two forms of option contracts such as call options and put
options. The call option enables the buyer to buy and the put option would
provide the seller the right to sell the underlying asset or instruments.
TOPIC 1 INTRODUCTION TO DERIVATIVES MARKETS  3

(d) Swap Contract


A swap contract is a transaction between two parties which simultaneously
exchanges cash-flows based on a notional amount of the underlying asset
(Bacha, 2012). The rate at which the amounts are exchanged is
predetermined according to either a fixed rate or an amount to be based on
a reference or benchmarking rate.

1.2 EVOLUTION OF DERIVATIVES


Similar to all other products, derivatives evolved through innovation in response
to growing demands of businesses. This evolution can be simplified as in Figure
1.2.

Figure 1.2: The evolution of derivative instruments


Source: Bacha (2012)

As you can see in Figure 1.2, forward contracts are probably the first derivative
instrument. Forward contracts tend to mitigate price risk between two parties.
Let us look at an illustration that demonstrates this type of contract.

Illustration 1.1:
A commodity producer is afraid of fall in prices when his commodity is ready in
future, while a consumer is fearful of an increase in prices in future. Both parties
meet, negotiate and agree on a price at which the transaction can be carried out at
the future date, thus a forward contract is made. The benefit of this contract is
that both parties have eliminated price risk by locking in their price or cost.
4  TOPIC 1 INTRODUCTION TO DERIVATIVES MARKETS

However, the forward contract has inherently three limitations (see Table 1.1).

Table 1.1: Three Limitations of Forward Contract

Limitations Explanations
Multiple Both parties should have opposite needs with respect to the
coincidence underlying asset, and matching timing and quantity.
Unfair pricing In forward contract, the price is reached through negotiation.
Stronger bargaining position of one party may lead to imposition of
the price.
Counterparty Though it is a legally binding contract, the recourse is slow and
risk costly. This increases the default risk in forward contract.

As these shortcomings of forward contracts became apparent, the need for


futures contract is developed.

As stated before, futures contract is essentially a standardised forward contract


traded on an exchange. In futures contract, the problems are addressed in these
ways:

(a) Multiple coincidences are resolved via exchange trading. Buyers and sellers
would transact in the futures contract closest to needed maturity and in as
many contracts as needed to fit the underlying asset size.

(b) Unfair pricing is resolved since each party is a price taker on the exchange
with the futures price being that which prevails in the market at the time of
contract initiation.

(c) Counterparty risk is overcome via the exchange acting as the intermediary
guarantees each trade by being the buyer to each seller and seller to each
buyer.

However, futures while overcoming flaws of forwards were inadequate for later
day business needs. Futures enabled hedging against unfavourable price
movement, but being locked-in also meant that one could not benefit from
subsequent favourable price movements. Therefore, this precise inadequacy is
addressed by option contracts.
TOPIC 1 INTRODUCTION TO DERIVATIVES MARKETS  5

Option contracts have three marked benefits over its predecessors which are:

(a) Options provide cover against both upward and downward movement of
asset prices.

(b) They are extremely flexible and can be combined to achieve different
objectives/cash flows.

(c) Only options can handle complicated business situations which cannot be
done by futures and forwards contracts.

1.2.1 Advent of Swaps


Are you aware that swaps are one of the fastest growing categories of
derivatives? What do swaps mean?

Swaps are customised bilateral transaction where both parties agree to


exchange cash flows at periodic intervals.

Being customised in nature, swap contracts are over-the-counter instruments.


There are four types of swaps that you need to know about. They are described
in Table 1.2.

Table 1.2: Four Types of Swaps

Swap Definition
Currency swaps Parties exchange one currency for another.
Commodity swaps Both parties exchange cash flows based on an underlying
commodity index or total return of a commodity in exchange for
a return based on a market yield.
Equity swaps It constitutes an exchange of cash flows based on different equity
indices.
Interest rate swaps It involves exchange of cash flows based on two different interest
rates.
6  TOPIC 1 INTRODUCTION TO DERIVATIVES MARKETS

SELF-CHECK 1.1
Define

(a) Forward contract; (c) Options contract; and

(b) Futures contract; (d) Swap contract.

1.3 EXCHANGE TRADED DERIVATIVES


What are exchange traded derivatives?

Exchange traded derivatives are listed and traded in an official exchange.


They are of standard contract size, maturity, delivery process and in the case
of commodity derivatives also of standard quality.

In exchange traded derivatives, the exchange becomes the intermediary between


buyers and sellers and guarantees the contract. Exchange trading shifts the
counterparty risk to the exchange. Other benefits include enhanced liquidity via
increased trading volumes which reduce transaction costs and price discovery,
for example, futures and option contracts.

1.4 OVER-THE-COUNTER (OTC) DERIVATIVES


What do over-the-counter (OTC) derivatives mean?

Over-the-counter (OTC) derivatives are customised transactions between


parties in a bilateral arrangement.

All elements of the transaction are negotiable, including pricing which usually
happens between corporate clients and financial institutions. Let us look at an
illustration that demonstrates this type of derivatives.
TOPIC 1 INTRODUCTION TO DERIVATIVES MARKETS  7

Illustration 1.2:
An exporter expects to receive foreign currency payment. Fearing a potential
depreciation of the currency, the exporter would want to hedge its position by
using currency derivatives like forward or swaps. The counterparty for this
hedging may be a financial institution.

Being a customised and bilateral transaction, a counterparty or default risk by


either party is possible in OTC.

1.5 EXCHANGE TRADED DERIVATIVES VERSUS


OVER-THE-COUNTER DERIVATIVES
What are the advantages and disadvantages of both exchange traded derivatives
and over-the-counter derivatives? Let us find out the answers in Table 1.3.

Table 1.3: Advantages and Disadvantages of Exchange


Traded versus Over-the-counter Contracts

Over-the-Counter (OTC) Exchange Traded


Advantages Disadvantages Advantages Disadvantages
Customisation is More costly Lower Customisation is not
possible transaction cost possible
Can fully hedge Not liquid Very liquid Some basis risks arising
a position from mismatch in asset,
quality, maturity and size
are possible
Can be altered Cannot be Can be easily -
for a particular reserved/offset reserved
need
- Potential for Market clearing -
„unfair‰ price prices
- Little transparency - -
in the pricing
- Counterparty risk No counterparty -
risk
8  TOPIC 1 INTRODUCTION TO DERIVATIVES MARKETS

SELF-CHECK 1.2

State the differences between exchange traded and over-the-counter


contracts.

1.6 MAIN PLAYERS IN DERIVATIVES MARKET


Lastly, let us look at the main players in the derivatives market (refer to
Table 1.4).

Table 1.4: Three Main Players of Derivative Markets

Trades Purpose
Hedgers  Players whose objective is risk reduction.
 Use derivative markets to manage or reduce risks.
 Usually businesses who want to offset exposures resulting from their
business activities.
Speculators  Players who establish positions based on their expectations of future
price movements.
 Take positions in assets or markets without taking offsetting
positions, expecting market to perform according to their
expectation.
Arbitrageurs  Players whose objective is to profit from pricing differentials
mispricing.
 Usually, they are looking for price divergences of the same financial
asset or instruments by closely monitoring the quoted prices.
 If there is price divergence that is sufficient to make some profits,
then they would purchase at the market with the lower price and sell
at the market with higher price.

Now let us look at societal impact of different categories of market players:

(a) Hedging enables businesses to plan better, and reduction in fluctuation of


their product prices can help reduce costs. This reduction in costs is passed
on to consumers in the form of lower prices.

(b) Arbitrageurs, by means of their activities, ensure no divergence exists


between different markets (spot, futures, options) for same asset classes.
Arbitrage activity enhances the price discovery process. For example,
arbitrage between markets in different countries ‰internationalises‰
TOPIC 1 INTRODUCTION TO DERIVATIVES MARKETS  9

product prices. This forces less efficient producers to enhance productivity


in order to remain competitive in business.

(c) Speculative activities are considered disruptive and create inefficiencies in


the market. But the enhanced volume due to speculators reduces
transaction costs and increases liquidity.

SELF-CHECK 1.3

Who are the main players in derivatives markets?

Ć The four common derivative instruments are forward contract, futures


contract, option contract and swap contract.

Ć The derivatives evolved through innovation in response to growing demands


of businesses. It started out with forward, futures and options.

Ć Exchange traded derivatives are listed and traded on an official exchange.


They are of standard contract size, maturity, delivery process and in the case
of commodity derivatives also of standard quality.

Ć An over-the-counter (OTC) derivative is a customised transaction between


parties in a bilateral arrangement. All elements of the transaction are
negotiable, including pricing.

Ć We can compare exchanged traded derivatives and over-the-counter


derivatives through their advantages and disadvantages. For example, over-
the-counter has the advantage of customisation but is more costly while
exchange traded has the advantage of lower transaction cost but is not
possible to customise.

Ć The main players of derivative markets are hedgers, speculators and


arbitrageurs.
10  TOPIC 1 INTRODUCTION TO DERIVATIVES MARKETS

Arbitrageurs Hedgers

Exchange traded derivatives Option contracts

Financial derivatives Over-the-counter (OTC) derivatives

Forward contracts Speculators

Futures contracts Swaps

Bacha, O. I. (2012). Financial derivatives: Markets and applications in Malaysia


(3rd ed.). Shah Alam, Malaysia: McGraw-Hill.
T op i c  Mechanics of
2 Futures
Markets
LEARNING OUTCOMES
By the end of the topic, you should be able to:
1. Identify the specification of a futures contract;
2. Explain the convergence of futures price to spot price;
3. Differentiate between forward and futures contracts;
4. Explain the mechanics of futures and operation of margins; and
5. Interpret newspaper quotes.

 INTRODUCTION
Did you know that contract specifications are the ground rules by which a
derivative contractÊs trading is dictated? What is the objective of a contract
specification? The objective of a contract specification is to lay out in clear and
uncertain terms the features and trading rules of the contract. This is to ensure
fairness to all parties involved and a clear and transparent process in settlement,
margining and so on.

Therefore, this topic is dedicated to the mechanics of futures markets which


consist of the specification of futures contract, the convergence of futures price to
spot, forward and futures contracts, the mechanics of futures and operation of
margins and newspapers quotes. So are you ready to embark this journey? Let us
continue the lesson!
12  TOPIC 2 MECHANICS OF FUTURES MARKETS

2.1 SPECIFICATION OF FUTURES CONTRACT


Did you know that there are different types of contract specifications for different
futures contracts? In this subtopic, we will be using BMD FTSE Bursa Malaysia
KLCI Futures (FKLI) as an example to explain the specifications of futures
contracts. Let us look at the following Table 2.1.

Table 2.1: Specifications of BMD FTSE Bursa Malaysia KLCI Futures (FKLI)

Exchange Bursa Malaysia Derivatives (BMD)


Product symbol FKLI
Underlying FTSE Bursa Malaysia Kuala Lumpur Composite Index (FBM
instrument KLCI)
Contract size FBM KLCI multiplied by RM50
Pricing unit Ringgit Malaysia
Minimum price 0.5 index point valued at RM25
fluctuation
Contract months Spot month, the next month and next two calendar quarterly
months. The calendar quarterly months are March, June,
September and December
Trading hours  1st session: 8:45 am to 12:45 pm
 2nd session: 2:30 pm to 5:15 pm
Daily price limit Twenty per cent per trading session for the respective contract
months except the spot month contract. There shall be no price
limits for the spot month contract. There will be no price limit for
the second month contract for the final five business days before
expiration
Final settlement Cash settlement based on the final settlement value
Final trading day The last business day of the contract month
Final settlement The final settlement value shall be the average value, rounded to
value the nearest 0.5 of an index point (values of 0.25 or 0.75 and above
being rounded upwards), taken at every 15 seconds or at such
intervals as may be determined by the exchange from time to
time from 3.45:30 pm to 4.45:15 pm plus one value after 5pm of
the FBM KLCI on the final trading day excepting the three
highest and three lowest values
Speculative Maximum number of net long or net short positions to be held:
position limit  10,000 contracts for all months combined

Source: http://www.bursamalaysia.com/market/derivatives/products/equity-
derivatives/ftse-bursa-malaysia-klci-futures-fkli/
TOPIC 2 MECHANICS OF FUTURES MARKETS  13

SELF-CHECK 2.1

State the specifications of a futures contract.

2.2 CONVERGENCE OF FUTURES PRICE TO


SPOT PRICE
We can safely say that as the delivery month of a futures contract approaches, the
price of the futures contract will close or even equal to the spot price. This is a
general observation that will occur regardless of the underlying asset of the
futures contract. It will happen because of arbitrage activities and the law of
supply and demand.

For instance, suppose the futures contract for crude palm oil (CPO) is higher than
the spot price when the delivery month of contracts approaches. In this case,
derivatives market players will have the arbitrage opportunity of shorting
(selling) the futures contracts and by going long (buying) the underlying asset ă
CPO ă and then making the delivery. By doing so, the player (trader) can lock in
profit since the amount of money received in selling the futures contract is more
than the amount of money buying the underlying asset to cover the position.

In the case of supply and demand, the effect of arbitrageurs shorting futures
contracts causes a drop in futures prices because it creates an increase in
the supply of contracts available for trade. Subsequently, buying the underlying
asset will cause an increase in the overall demand for the asset and the spot price
of the underlying asset will increase as a result.

As arbitragers continue to do this, the futures prices and spot prices will slowly
converge until they are more or less equal. The same sort of effect occurs when
spot prices are higher than futures except that arbitrageurs would short (sell) the
underlying asset and go long (buy) the futures contracts.
14  TOPIC 2 MECHANICS OF FUTURES MARKETS

2.3 FORWARD AND FUTURES CONTRACTS


Variants of modern day forward contracts were present in the 17th century
amongst rice farmers and dealers in Japan (Bacha, 2012). In the US and Europe,
forward contracts seem to have their beginning in agriculture-based businesses.
This is because agriculture has multiple factors beyond human control such as
weather, diseases, insects and so on. All of these factors directly impact the
supply chain, thus, the pricing. Price risk directly affects business planning for
mills and food processing industries. Let us look at an illustration to demonstrate
this situation.

Illustration 2.1:
Assume there are two parties, a cocoa farmer who has planted crop and expects
to harvest in six months. The second party is a confectioner who needs cocoa,
assuming he has enough inventory for six months.

Both parties are exposed to price risk:

(a) The cocoa farmer has a risk of spot prices falling between now and six
months. Any decrease in prices would result in lower profits for the cocoa
farmer or a sharp loss may result in outright loss.

(b) The confectioner has a risk of spot prices increasing in six months. Any
increment in price would result in increased costs and reduced profits.

Since both parties face a price risk, it is beneficial for them to go into an
arrangement that would protect them from this, thus, a forward contract. Under
the forward contract, the farmer would agree to deliver X amount of cocoa in six
months while the confectioner agrees to receive X amount of cocoa in six months.
The price, time and quantity are agreed mutually at the initiation of contract.
Both parties through this arrangement eliminate the uncertainty in prices.
Therefore, forward contract is used as a documented contract which legally binds
the parties.

Parties in a forward contract are said to have taken a long (buy) and short (sell)
position (refer to Table 2.2).

Table 2.2: Defections of Long and Short Positions

Long Position (Buyer) Short Position (Seller)


Is the party that promised to make Is the party that promised to make
delivery the confectioner? delivery in the illustration the cocoa
farmer?
TOPIC 2 MECHANICS OF FUTURES MARKETS  15

So what can we conclude about forward contracts?

A forward contract is a two staged process. The first step is negotiation and
agreement at the initiation of contract. The second step is the maturity date,
which is, when the contract is consummated and commodity and money
exchange hands.

These two stages are further elaborated in Figure 2.1.

Figure 2.1: Two stages of forward contract


Source: Bacha (2012)

However (as we discussed in Topic 1), there are three main problems associated
with forward contracts. Can you still recall? Therefore, we need something better
than forward contracts. Eventually, the futures contracts are the solution to this
limitation.

2.3.1 Futures Contracts


As stated before, futures contracts were introduced to address the problems in
forward contracts. What do futures contracts mean?

Futures contracts are standardised and exchange traded forward contracts.

Standardisation enables futures contracts to be traded on exchanges. There are


four key features that are standardised in this type of contract (see Figure 2.2).
16  TOPIC 2 MECHANICS OF FUTURES MARKETS

Figure 2.2: Four standardised key features in futures contracts

Now let us look at Table 2.3 which summarises the differences between futures
contracts and forward contracts.

Table 2.3: Forward Contracts versus Futures Contracts

Forward Contracts Futures Contracts


 Over-the-counter contract  Exchange traded, market prices
 Customised to the needs of  Standardised as per specifications of the
parties exchange
 Parties deal directly with each  Clearing house is intermediary
other
 No margin payments  Margins and marking to market processes
 Illiquid/untradeable  Highly liquid/easily tradeable
 Prices are negotiated

SELF-CHECK 2.2

Differentiate between forward and futures contracts with one


example each.

2.4 MECHANICS OF FUTURES AND


OPERATION OF MARGINS
This subtopic will discuss further mechanics of futures and operation of margins.
More importantly, it exposes you to the concept of margining and the process of
marking-to-market process.
TOPIC 2 MECHANICS OF FUTURES MARKETS  17

2.4.1 Mechanics of Futures


In order to understand the mechanism of future trading, we will carry on with
the earlier cocoa farmer and confectioner example (Illustration 2.1).

Contract specifications:

(a) Assume the farmer expects to produce 120 tons of cocoa in six months.

(b) The futures contracts are available in 3, 6, 9 and 12 months maturity with a
standard size of 10 ton per contract.

(c) The current quoted price for six month cocoa futures is RM100 per ton,
translating into RM1,000 per contract.

(d) For simplicity purposes, assume confectioner also requires 120 tons of
cocoa in six months.

The hedging mechanism would operate as follows:

(a) The cocoa farmer would call his broker and sell (short) 12 contracts of six
month cocoa futures. The farmer now knows he will receive RM12,000
(RM1,000  12) in six months.

(b) The confectioner will call his broker and buy (long) 12 contracts of six
month cocoa futures. The confectioner now knows he has to pay RM12,000
(RM1,000  12) in six months.

(c) Neither parties need to know who the counterparty is and are assured of
the delivery/payment of the trade.

(d) On registration of the trade, the clearing house guarantees the performance
of the contract.

On day 180, the contract will be settled via the following process (see Figure 2.3).
18  TOPIC 2 MECHANICS OF FUTURES MARKETS

Figure 2.3: Settlement of a futures contract


Source: Bacha (2012)

Again, the mechanics of futures contracts overcome the problems faced by


forward contracts as stated in Table 2.4.

Table 2.4: Overcoming of Problems of Forward Contracts

Problem in
Alternative Provided by Futures Contracts
Forward Contracts
Multiple  All contracts are traded on the exchange, so it becomes the
coincidence focal point for all buyers and sellers, making finding the
counterparty simple.
 Standardised sizes allow divisibility, that is, the seller does
not need to find a counterparty with the exact matching
quantity requirements.
Unfair pricing  Through interaction of multiple buyers and sellers in the
exchange, market clearing prices are achieved.
 These prices would reflect currently available information
and demand-supply conditions.
Counterparty risk  Futures exchange through its clearing house becomes the
counterparty for every trade.
 Clearing house guarantees each trade.
 In case of default of one party, clearing house stands ready to
complete the trade.
TOPIC 2 MECHANICS OF FUTURES MARKETS  19

2.4.2 Margining and Marking-to-Market


Keep in mind that the clearing house in guaranteeing every trade essentially
transfers the default risk on itself. The clearing house manages its own risk by
means of margin and marking-to-market. What is the marking-to-market
process?

Marking-to-market is a process where the clearing house recognises


gains/losses incurred by the long and short positions and adjusts their
margin accounts accordingly. It is done on a daily basis.

There are two types of margins maintained with the clearing house (see Table
2.5).

Table 2.5: Two Types of Margins

Type Description
Initial margin  It has to be deposited on the day the futures contract is entered.
 This is done since both long and short position can potentially lose
with variation of prices.
 Typically 10 to 20 per cent based on credit worthiness.
Maintenance  It is the additional margin payments that would have to be paid if
margin the initial margin falls below a certain level.
 It is usually a per cent of the initial margin; for example 70 per cent
of initial margin.
 A margin call is a requirement that margin account be brought
back to its initial margin level by paying an additional margin.

Illustration 2.2:
Let us look at Table 2.6 for the summary of the process of marking-to-market on
the cocoa farmer and confectioner.
20  TOPIC 2 MECHANICS OF FUTURES MARKETS

Table 2.6: Process of Marking-to-Market

At the time of initiation of contract, both parties would be required to post the
initial margin of 10% = RM1,200 (10% of RM12,000).
Action Maintenance margin for both parties would be RM840 (70% of initial margin).
If either partyÊs margin balance falls below RM840, he would receive a margin
call from his broker.
On Day 1, the futures price falls to RM98 per ton.
A fall in price would profit the short position but work against the long
position. This represents a movement of funds out of the ‰losing‰ account and
Day 1
into the ‰gaining‰ account.
The adjustment amount would be RM240 (RM2.00  (10 tons per contract 
12 contracts)).
On Day 2, the futures price falls to RM97 per ton.
The adjustment amount would be RM120 (RM1.00 ï (10 tons per contract ï
12 contracts)).
Day 2 RM120 is deducted from the long position and the same amount added to the
short position.
After the deduction on Day 2, the long positionÊs margin balance is RM840
(RM1,200 ă RM240 (Day 1) ă RM120 (Day 2)).
On Day 3, the futures price rises to RM98 per ton.
The adjustment amount would be RM120 (RM1.00 ï (10 tons per contract 
Day 3 12 contracts)).
RM120 is deducted from the short position and the same amount added to the
long position.
On Day 4, the futures price falls to RM96 per ton.
The adjustment amount would be RM240 (RM2.00  (10 tons per contract ï
12 contracts)).
Day 4 RM 240 is deducted from the long position and the same amount added to the
short position.
After the deduction on Day 4, the long positionÊs margin balance is RM720
(RM1,200 ă RM240 (Day 1) ă RM120 (Day 2) + RM120 (Day 3) ă RM240 (Day
4)).
The long position will receive a margin call from his broker, requiring him to
pay an additional RM480 (RM1,200 ă RM720) to top up to the level of initial
Action margin.
This amount has to be paid within a stipulated time the next day. Failure to do
so will result in the defaulting of the long position.
TOPIC 2 MECHANICS OF FUTURES MARKETS  21

The process in Table 2.6 can be simplified into Figure 2.4.

Figure 2.4: Margins and the marking-to-margin process


Source: Bacha (2012)

Thus, marking-to-market is exercised to ensure players pay up their losses as


they occur. With loses already paid for the parties, they would not have any
incentive to default.

Now let us look at an example.

Example 2.1:
If the spot price on the maturity date is RM80 per ton, the confectioner will have
an incentive to default because the confectioner can buy the same product from
open market at RM80 per ton. With marking-to-market on daily basis, he would
have paid RM2,400 (RM20 per ton) to the margin account. The confectioner
would have to pay RM80 per ton at maturity to take delivery of the cocoa.

SELF-CHECK 2.3

Describe the operation mechanisms of a futures contract.


22  TOPIC 2 MECHANICS OF FUTURES MARKETS

2.5 NEWSPAPER QUOTES


Lastly, let us look at newspaper quotes. For your information, commodity futures
generally have a few basic columns as quoted in newspaper like the Wall Street
Journal. These quotes are shown in Table 2.7.

Table 2.7: Newspaper Quotes

Quote Meaning
Open This is simply the opening price.
High The high price for the contract over the course of the trading session.
Low The low price for the contract over the course of the trading session.
Settle The closing price at the end of the trading session.
Change The change between the closing price at the end of the trading session and
the closing price of the previous trading session.
Lifetime The highest and lowest price that the contract has ever traded at.
high/low
Open The number of open positions. A seller and a buyer of contract taken
interest together make one open position (simply because there cannot be a
futures contract trading without both a buyer and a seller).

Now let us look at Figure 2.5 for an example.


TOPIC 2 MECHANICS OF FUTURES MARKETS  23

Figure 2.5: Newspaper quotes of a futures contract


Source: Beattie (2015)

Take note that the index futures are basically identical to commodities except
that they use a multiplier in the pricing. Here are some index futures from
Bloomberg (see Figure 2.6).
24  TOPIC 2 MECHANICS OF FUTURES MARKETS

Figure 2.6: Newspaper quotes of index futures


Source: Beattie (2015)

Ć There are different types of contract specifications for different futures


contracts. Some specification of futures contract are underlying instruments,
contract size, contract months, trading hours and many more.

Ć As the delivery month of a futures contract approaches, the price of the


futures contract will close or even equal to the spot price. This is a general
observation that will occur regardless of the underlying asset of the futures
contract. It will happen because of arbitrage activities and the law of supply
and demand.

Ć A forward contract is a two staged process. The first step is negotiation and
agreement at the initiation of contract. The second step is the maturity date
which is when the contract is consummated and commodity and money
exchange hands.

Ć Futures contracts are standardised and exchange traded forward contracts.


Standardisation enables futures contracts to be traded on exchanges.

Ć The mechanics of futures contracts overcome the problems faced by forward


contracts which are multiple coincidence, unfair pricing and counterparty
risk.
TOPIC 2 MECHANICS OF FUTURES MARKETS  25

Ć Certain newspaper quotes that you should be familiarised are „open‰,


„high‰, „low‰, „settle‰, „change‰ and so on.

Contract specifications Marking-to-market

Forward contracts Mechanics of futures

Futures contracts Newspaper quotes

Margining

Bacha, O. I. (2012). Financial derivatives: Markets and applications in Malaysia


(3rd ed.). Shah Alam, Malaysia: McGraw-Hill.

Beattie, A. (2015, January 15). A quick guide for futures quotes. Investopedia.
Retrieved from http://www.investopedia.com/articles/active-trading/
011515/quick-guide-futures-quotes.asp#ixzz48DAXa5Fp

Phung, A. (n. d). Why do futuresÊ prices converge upon spot prices during the
delivery month? Investopedia. Retrieved from http://www.investopedia.
com/ask/answers/06/futuresconvergespot.asp#ixzz488KIwgeB
Topic  Determination
3 of Forward
and Futures
Prices
LEARNING OUTCOMES
By the end of the topic, you should be able to:
1. Differentiate between investment and consumption assets;
2. Define short selling;
3. Measure interest rates;
4. Determine the forward and futures prices for investment assets;
5. Find the value of forward contracts; and
6. Compare forward and futures prices.

 INTRODUCTION
In the previous topic, we were familiarised with the definitions of forward and
futures contracts and their operating mechanisms. Now, what will we cover in
this new topic?

In this new topic, we are going to differentiate between investment and


consumption assets, define short selling and determine the forward prices. Last
but not the least, you will learn how to value the forward contracts. Are you
ready to discover more? Let us continue the lesson.
TOPIC 3 DETERMINATION OF FORWARD AND FUTURES PRICES  27

3.1 INVESTMENT ASSETS VERSUS


CONSUMPTION ASSETS
What is the major difference between investment assets and consumption assets?
Do you have any idea? Let us find out the answer in Table 3.1.

Table 3.1: Investment Assets versus Consumption Assets

Investment Assets Consumption Assets


Assets held by significant numbers of Assets held primarily for consumption
people purely for investment purposes such as copper and oil.
such as stocks, bonds, sukuk, gold and
silver.

How do we determine the forward and futures prices of an investment asset? We


can determine the forward and futures prices of an investment asset by using
arbitrage arguments. However, we cannot do this for consumption assets (Hull,
2012).

SELF-CHECK 3.1

Differentiate between consumption and investment assets.

3.2 SHORT SELLING


Did you know that it is important to define short selling before we go into the
determinations of forward and futures prices? What does short selling mean?

Basically, short selling is selling the securities you do not own.

Let us say, your broker borrows financial assets from another customer and sells
them in the market in the usual way. At some stage you must buy the securities
back so they can be replaced in the account of the client. You must pay dividends
and other benefits the owner of the securities receives.

What are the strategies of short selling? Let us find out the answer in Table 3.2.
28  TOPIC 3 DETERMINATION OF FORWARD AND FUTURES PRICES

Table 3.2: Strategies of Short Selling

Aim Strategy


If you believe that a stock is going to go up in price, go long (buy).
If your prediction is right, you will get profits ă strategy is „buy low and sell
high‰.


If you think that a stock is going to go down in price, go short (sell).
Again if your prediction is right, you gain profit ă strategy is „sell high and
buy low‰.

Now let us look at an example of short selling in Figure 3.1.

Figure 3.1: Example of short selling


Source: Hull (2012)

SELF-CHECK 3.2

Define short selling.


TOPIC 3 DETERMINATION OF FORWARD AND FUTURES PRICES  29

3.3 MEASURING INTEREST RATES


Firstly, what does interest rate mean? According to Gitman and Zutter (2015), the
term interest rate usually applies to debt instruments such as bank loans or
bonds; the compensation paid by the borrower of funds to the lender; from the
borrowerÊs point of view, the cost of borrowing funds.

The unit measurement of an interest rate is the compounding frequency. Due to


the compounding effects (monthly, quarterly, semi-annual and annual),
compounding interest rates are different too. This means that the effective
interest rate is different from the nominal interest rate. We derive the
continuously compounded interest rates by compounding more and more
frequently as described in the limit. For example:

(a) RM1,000 grows to RM1,000 erT when invested at a continuously


compounded rate r for time t.

(b) RM1,000 received at time t discounts to RM1,000 erT at time zero when the
continuously compounded discount rate is r.

The annual rate of interest actually paid or earned is called effective (true) annual
rate (EAR). In principle, the effective rate is bigger than the nominal rate
whenever compounding happens more than once per year. We can use the
following formula to compute the effective (true) annual rate (EAR).

r m
EAR  (1  ) 1
m

3.4 FORWARD AND FUTURES PRICES FOR


INVESTMENT ASSETS
Now let us look at forward and futures prices for investment assets. In other
words, we are going to look at how the forward and futures prices are priced.

3.4.1 Forward Price for Investment Assets


Did you know that the easiest forward contract to value is one written on an
investment asset that provides the holder with no income? Let us take the crude
palm oil (CPO) example. By using it, we are demonstrating the forward price of
investment assets.
30  TOPIC 3 DETERMINATION OF FORWARD AND FUTURES PRICES

Example 3.1:
Find the forward price of investment assets for crude palm oil (CPO).

Solution:
F0 = S0 (1 + r)T
(Assuming no storage costs)
If r is compounded continuously instead of annually:

F0 = S0 erT

Suppose CPO is currently RM3,000 per ton and the risk-free rate is 6 per cent per
annum (p.a.), a 6-month forward contract on CPO will have a price:

F0 = S0 erT = RM3,000  e0.061/2 = RM3,091.36

Suppose CPO is currently RM3,000 per ton and the risk-free rate is 6 per cent per
annum and the 6-month forward contract on CPO has a price F0 = RM3,100, then
the strategy will be (refer to Table 3.3):

Table 3.3: Strategy for Forward Price for Investment Assets on a Price F0 = RM3,100

Position Today Position at Maturity


 Borrow RM3,000  Sell one ton of CPO for RM3,100
 Buy one ton of CPO through the forward contract
 Short (sell) a forward contract on one  Repay RM3,000 loan with RM3,091.36
ton of CPO  The profit will be RM3,100 ă
RM3,091.36 = RM8.64 for per
contract

Suppose CPO is currently RM3,000 per ton and the risk-free rate is 6 per cent per
annum and the 6-month forward contract on gold has a price F0 = RM3,050, then
the strategy will be (refer to Table 3.4):

Table 3.4: Strategy for Forward Price for Investment Assets on a Price F0 = RM3,050

Position Today Position at Maturity


 Sell one ton of CPO  Collect RM3,091.36 on the RM3,000
 Invest the RM3,000 at 6 per cent investment
 Take a long position (buyer) in a  Buy one ton of CPO for RM3,050
forward contract on one ton of CPO through the forward contract
TOPIC 3 DETERMINATION OF FORWARD AND FUTURES PRICES  31

3.4.2 Futures Prices for Investment Assets


The next question is how the futures contract is priced. The starting point would
be the spot price of the underlying asset. Futures price has to be determined by
adjusting the spot price of the underlying asset by the carrying cost. Since a
futures contract calls for delivery at a future date, the short position (seller) will
have to incur additional costs like storage, handling costs and so on. Since the
payment for the futures will be received only at maturity, the short position also
incurs the opportunity cost of later payments.

With the stated logic, the price determination of futures is based on the current
spot price of the underlying asset adjusted for costs. Cost of storage includes the
costs of handling, spoilage, shrinkage and so on. The opportunity cost of having
to receive payment is only at maturity of the futures contract. The price needs to
adjust for „convenience yield‰. What does convenience yield refer to?

Convenience yield refers to any benefits that could accrue to the short
position (seller) from holding onto the spot asset until maturity.

According to Bacha (2012), the cost of carry model (COC) can be calculated by
using this formula:
t,T
F = S (1 + rf + c ă y)
t,T 0

Where:

F = Futures price for a contract with maturity from t to T (t = today, T


t,T
= maturity)

S = Current spot price of the underlying asset


0

rf = Annualised risk free interest rate (being the proxy for the
opportunity cost of later payment)

c = Annualised storage cost in per cent (inclusive of shipping


handing, shrinkage, spoilage etc.)

y = Convenience yield (annualised percentage)


32  TOPIC 3 DETERMINATION OF FORWARD AND FUTURES PRICES

For our example of CPO, the following information is given:

(a) Spot price of CPO = RM3,000 per ton

(b) Risk free interest rate (rf) = 6 per cent annualised

(c) Storage cost = RM50.00 per ton/year

(d) Convenience yield to seller = nil

The correct price of a 180 day (six months) futures contract according to the cost
of carry model is:
t,T
F = S (1 + rf + c ă y)
t,T 0
0.5
F = RM3,000 (1 + 0.06 + 0.017 ă 0)
180
0.5
= RM3,000 (1.077)
= RM3,000 (1.037786)
= RM3,113.36

Take note that the storage cost of RM50 per ton per year is entered as a
percentage in the equation (50/3,000).

The equation is raised to the power of 0.5. This is to denote the six months or half
year period (1/2).

3.5 VALUING FORWARD CONTRACTS


Now let us learn how to value forward contracts. Let us say:

K is delivery price in a forward contract


F is forward price that would apply to the contract today
0

The value of a long forward contract, ”, is


-rT
f = (F 0 ă K )e

Similarly, the value of a short forward contract is:


-rT
f = (K ăF0 )e
TOPIC 3 DETERMINATION OF FORWARD AND FUTURES PRICES  33

Let us use an example of the forward contract on the ABC bond.

Example 3.2:
Suppose that we had taken a long position in the contract on August 2, 2015 at
RM984.27. Ten minutes later, interest rates have risen to 8 per cent per annum.
Have we made or lost money?

Solution:
Let us say, we have agreed to pay RM984.27 on August 2, 2015 for an asset that
will be worth RM970.05. This will be a loss of RM14.22 at expiry.
-rT
f = (F 0 ă K )e

= (970.05 ă 984.27)e-0.08

= RM13.13

The present value of that loss is RM13.13.

3.6 ARE FORWARD PRICES AND FUTURES


PRICES EQUAL?
Generally, we assume that the prices of forward and futures contracts are the
same (Hull, 2012). In the case where the interest rates are uncertain, they are
slightly different from a theoretical perspective:

(a) If there is a strong positive correlation between the asset price and interest
rates, then the futures price is slightly higher than the forward price.

(b) In the case of the strong negative correlation, then the futures price is
slightly lower than the forward price.

In practice, as additional factors, we also must take into account the probability
of counterparty default, taxes, transactions costs and the treatment of margins.
Keep in mind that it would be dangerous to ignore these differences. For
example, for long-lived contracts like the Euro dollar, futures contracts come
with maturities as long as 10 years.

SELF-CHECK 3.3

How do we determine whether forward and futures prices are equal?


34  TOPIC 3 DETERMINATION OF FORWARD AND FUTURES PRICES

 Investment assets are assets held by a significant number of people purely for
investment purposes. Some examples of these assets are stocks, bonds, sukuk,
gold and silver.

 Consumption assets are assets held primarily for consumption such as


copper and oil.

 Basically, short selling is selling the securities you do not own.

 According to Gitman and Zutter (2015), the interest rate is the compensation
paid by the borrower of funds to the lender; from the borrowerÊs point of
view, the cost of borrowing funds. It is usually applied to debt instruments
such as bank loans or bonds. The unit measurement of an interest rate is the
compounding frequency.

 Forward prices for investment assets can be determined by using:

F0 = S0 (1 + r)T

 Futures prices for investment assets can be calculated by using:


t,T
F = S (1 + rf + c ă y)
t,T 0

 Value of forward contracts can be determined by using:

f  (F 0  K )e rT for a long forward contract

f  (K  F 0 )e  rT for a short forward contract

 Generally we assume that the prices of forward and futures contracts are the
same (Hull, 2012). In the case where the interest rates are uncertain, they are
slightly different from a theoretical perspective:

 If there is a strong positive correlation between the asset price and interest
rates, then the futures price is slightly higher than the forward price.

 In the case of the strong negative correlation, then the futures price is
slightly lower than the forward price.
TOPIC 3 DETERMINATION OF FORWARD AND FUTURES PRICES  35

Consumption asset Measurement of interest rates

Forward price for investment assets Short selling

Futures price for investment assets Valuing forward contracts

Investment asset

Bacha, O. I. (2012). Financial derivatives: Markets and applications in Malaysia


(3rd ed.). Shah Alam, Malaysia: McGraw-Hill.

Gitman, L. J., & Zutter, C. J. (2015). Principles of managerial finance (7th ed.).
Essex, England: Pearson Education.

Hull, J. C. (2012). Options, futures, and other derivatives (8th ed.). Boston, MA:
Prentice Hall.
Topic  Hedging
4 Strategies
Using Futures
LEARNING OUTCOMES
By the end of the topic, you should be able to:
1. State the basic principles of hedging;
2. Explain the arguments for and against hedging;
3. Determine basis risk and minimum variance hedge ratio;
4. Apply the stock index futures (SIF) contract; and
5. Summarise single stock futures (SSF).

 INTRODUCTION
In this topic, we are going to elaborate on basic principles of hedging as well as
the arguments for and against hedging. At the same time, we are going to
describe the basis risk and minimum variance hedge ratio. Last but not least, we
also need to look at the characteristics of stock index futures (SIF) and single
stock futures (SSF).
TOPIC 4 HEDGING STRATEGIES USING FUTURES  37

4.1 BASIC PRINCIPLES OF HEDGING


What is the basic question of hedging? The basic question of hedging is how to
take a position that reduces or neutralises the risk as much as possible. There are
two different situations that you will face; whether you are optimistic about the
market or you are bearish about the market. Put differently, in one situation are
you going to buy a financial asset or you are going to sell one? In the situation
that you are going to purchase a stock, your position will be a long position.
Whereby, in the situation you are going to sell a stock, your position will be a
short position. In other words:

(a) A long futures hedge is applicable when you will buy a financial asset in
the future and are thinking of locking in the price today.

(b) A short futures hedge is more appropriate when you will sell a financial
asset in the future and are thinking of locking in the price today.

In the case of hedging, two parties who have opposite needs can eliminate risk
by offsetting risks. For instance, if a crude palm oil (CPO) producer and a
cooking oil producer enter into a forward contract, they are able to eliminate the
risk each other faces, in this case, the futures price of CPO. At the same time, the
hedgers can also transfer price risk to speculators so that the speculators can
absorb price risk from hedgers.

The next question is how many contracts you should hedge? Let us say, a farmer
will harvest 1,000 tons of CPO in six months. Then, the farmer wants to hedge
against a price decrease in CPO price. The CPO is quoted in Malaysian Ringgit
per ton at 10 tons per contract. It is currently at RM25,000 for a contract six
months out and the spot price is RM24,000. To hedge, you should sell 100 CPO
futures contracts by using this formula:

1 , 000 tons
 100 contracts
10 tons per contract

Now, you do not need to worry about the price of CPO since you have locked the
futures price.

In the case of currency, let us look at the following illustration in Table 4.1.
38  TOPIC 4 HEDGING STRATEGIES USING FUTURES

Table 4.1: Strategies of Currency Futures Contract

Situation Action
If you are going to PAY foreign currency in the Long position in a forward
future, agree to PURCHASE the foreign contract
currency.
If you are going to RECEIVE foreign currency in Short position in a forward
the future, agree to SELL the foreign currency. contract

ACTIVITY 4.1

You are a Malaysian importer of Chinese goods and have just ordered
next yearÊs inventory. Payment of RMB1 million is due in one year.
How can you fix the cash outflow in Malaysian ringgit?

4.2 ARGUMENTS FOR AND AGAINST


HEDGING
According to Hull (2012), the arguments in favour of hedging are readily
apparent while the arguments against hedging are somewhat more understated.
Let us look at Table 4.2 to understand more about this matter.

Table 4.2: Pro and Cons of Hedging Instruments

Category Description


Companies should focus on the main businesses they are in and should
take steps to minimise risks arising from:
 Interest rates;
Pro  Exchange rates; and
 Other market variables.


 Shareholders are usually well diversified and can make their own
hedging decisions:
 Not everyone agrees that a firm should hedge.
Con  Hedging by the firm may not add to shareholder wealth if the
shareholders can manage exposure themselves.
 Hedging may not reduce the non-diversifiable risk of the firm.
Therefore, shareholders who hold a diversified portfolio do not
help when management hedges.
TOPIC 4 HEDGING STRATEGIES USING FUTURES  39

 It may increase risk to hedge when competitors do not:


 Consider single competitor in a commodity industry dominated
by competitors who do not hedge.
 If the competitor hedges the raw materials and then prices of
raw materials fall, the price of the finished product will fall as
well ă this could decrease profit margins.
 Explaining a situation where there is a loss on the hedge and a gain
on the underlying can be difficult.

SELF-CHECK 4.1

1. Explain the basic principles of hedging.

2. Describe the arguments for and against hedging.

4.3 BASIS RISK


Did you know that it may be difficult to find a forward contract on the asset that
you are trying to hedge? This is because there may be uncertainty regarding the
maturity date. These problems give rise to basis risk. But firstly, what does basis
mean?

Basis is the difference between spot and futures.

In a simpler form,

Basis = Spot price of asset to be hedged ă Futures price of contract used

Basis risk arises because of the uncertainty about the basis when the hedge is
closed out. For example, variation of basis over time is as shown in Figure 4.1.
40  TOPIC 4 HEDGING STRATEGIES USING FUTURES

Figure 4.1: Variation of basis over time


Source: Hull (2012)

Now, let us look at the basis risk in long and short positions as shown in Table
4.3.

Table 4.3: The Basis Risk in Long and Short Positions

Long Hedge Short Hedge


F1 : Initial futures price F1 : Initial futures price
F2 : Final futures price F2 : Final futures price
S2 : Final asset price S2 : Final asset price

You hedge the futures purchase of an asset You hedge the futures sale of an asset by
by entering into a long futures contract entering into a short futures contract

Cost of asset = S2 ă (F2 ă F1) = F1 + Basis Price realised = S2+ (F1 ăF2) = F1 + Basis

4.4 MINIMUM VARIANCE HEDGE RATIO


How do we find out the hedge ratio? In order to find out the hedge ratio, we can
divide the size of the exposure by size of the position taken in the futures
contracts or simply stated:

Size of the exposure


Hedge ratio 
Size of the position taken in the futures contracts

If the objective of the hedger is to minimise risk, setting the hedge ratio equal to
one is not necessarily optimal.
TOPIC 4 HEDGING STRATEGIES USING FUTURES  41

Proportion of the exposure that should optimally be hedged is:

s
h*  
F

Where:

(a) s is the standard deviation of S, the change in the spot price during the
hedging period;

(b) F is the standard deviation of F, the change in the futures price during
the hedging period; and

(c)  is the coefficient of correlation between S and F.

The number of futures contracts required is

NA
N *  h *
QF

Where:

NA = Size of the position being hedged (units)


QF = Size of one futures contract (units)
N* = Optimal number of futures contracts

Let us look at an example.

Example 4.1:
Stock index futures (SIF) are used for hedging equity exposures by fund
managers. A fund manager has exposure to Malaysian stocks and intends to
keep his position in the stocks since he thinks the underlying fundamentals are
good. However, he is worried about volatility that could erode the current value
of his portfolio. How can he use SIF to hedge?

The following information is available to you today:

(a) Current value of portfolio = RM2,125,000;

(b) rf rate = 5% per year;


(c) Dividend yield on portfolio = 1.5% annualised;

(d) Spot index value = 1,700 points;


42  TOPIC 4 HEDGING STRATEGIES USING FUTURES

(e) Six months SIF futures contract = 1,715.00 points; and

(f) Assume the futures will expire in exactly 180 days.

Solution:
The fund manager has a long position in stocks, hedging would require
establishing an offsetting short position in SIF contracts. But how many SIF
contracts should he short? The answer depends on how closely the portfolio is
correlated to the market index. If it exactly tracks the FBM KLCI, then he can do a
base hedge.

Current ringgit value of portfolio


Base hedge 
Current ringgit value of index

2,125,000
 = 25 contracts
1,700  RM 50

According to base hedge the fund manager can short 25 SIF contracts.

Let us assume that the fund manager beta of portfolio is 1.20. The number of
contracts he requires to hedges is calculated via the following two methods (refer
to Table 4.4).

Table 4.4: Two Methods to Calculate the Number of Contracts to Hedge

Method 1 Method 2
Base hedge  Beta of portfolio No. of contracts =
25  1.2 = 30 contracts Current ringgit valu e of portfolio  Beta of portfolio
Ringgit va lue of index
2,125,000  1.2
= = 30 contracts
1,7000  50

Therefore, the fund manager goes short 30 contracts in SIF.

SELF-CHECK 4.2

Explain basis risk and minimum variance hedge ratio with one
appropriate example.
TOPIC 4 HEDGING STRATEGIES USING FUTURES  43

4.5 STOCK INDEX FUTURES (SIF) CONTRACT


What does a stock index futures (SIF) mean?

A stock index futures (SIF) contract is an exchange traded futures contract


which has its underlying asset as a basket of common stocks.

The basket of common stocks would together make up an index. For example,
FBM KLCI is an index comprising 30 stocks traded on Kuala Lumpur Stock
Exchange. A SIF contract entitles the holder to ‰take delivery‰ of the group of
stocks that make up the index at a pre-specified price and at a pre-determined
future date.

All SIF contracts are cash settled contracts. The need for cash settlement arises
from the fact that physical delivery of the group of stocks that make up an index
would be very cumbersome and costly.

First SIF was introduced in 1982 by Kansas City Board of Trade on the Valueline
Index. In Malaysia, FTSE Bursa Malaysia Kuala Lumpur Composite Index
Futures was launched on 15 December, 1995. Most popular SIF contracts globally
are:

(a) KOSPI Futures (Korea Stock Price Index);

(b) S & P 500 (Index of 500 US stocks);

(c) NSIF (Nikkei Stock Index Futures);

(d) FTSE 100 (index of 100 British stocks); and

(e) Valueline Index (US stocks).

4.5.1 Why Use Stock Index Futures (SIF) Contracts?


Why do we use the stock index futures (SIF) contracts? There are five main
reasons why we should use SIF as stated in Table 4.5.
44  TOPIC 4 HEDGING STRATEGIES USING FUTURES

Table 4.5: Advantages of Stock Index Futures (SIF)

Advantage Description
Diversification  Diversification benefits refer to reduction in risk as one diversifies
benefits across assets.
 This diversification benefit in SIF arises from the underlying
portfolio of stocks which constitutes the index. Thus, purchasing a
SIF contract is similar to buy each of the component stocks in the
index.
 According to portfolio theory, investment in a broad range of
stocks reduces unsystematic risk.
Lower  Brokerage costs like commissions are lower on a percentage of
transaction face value bases.
cost  The margins that need to be posted for SIF contracts are also much
lower relative to full payment on stock purchase.
 Transaction costs to buy each and every single stock in the index
separately are much higher than buying the stock index future.
Provides  A margin in SIF transactions means investment outlays that are
leverage much lower than transactions in the stock market.
 It implies that there would be automatic leverage with SIF
contracts.
Market  A position in SIF contracts allows for exposure to the entire
exposure and market, that is exposure to broad based market movements. For
stock selection example, suppose a Malaysian mutual fund manager is optimistic
about the Malaysian economy and wants exposure to Malaysian
stocks. He gets instant exposure to the overall market by going
long a FBM KLCI futures contract.
 In the absence of SIF contracts, the foreign fund manager would
have to engage in individual stock selection to assemble a
portfolio of Malaysian stocks.
 SIF contracts offer broad market exposure and are suitable for
passive investment strategies.
Hedging,  There are two ways in which SIF contracts are particularly suited
portfolio for hedging purposes:
insurance and  Managing systematic risk
risk
 Systematic risk is the risk that remains even after one has
management
put together a broad portfolio of assets.
 An investor can eliminate all unsystematic risk through
diversification but would still be faced with systematic
risk.
 Hedging the overall value of a portfolio
 SIF contracts provide a very effective, easy and low cost
method by which equity exposure could be hedged.
TOPIC 4 HEDGING STRATEGIES USING FUTURES  45

4.5.2 The Main Players in Stock Index Futures (SIF)


Markets
Who are the main players in SIF markets? The main players in SIF markets are
institutional investors largely due to the large money amounts involved in SIF
trading. For FBM KLCI futures contracts, the multiplier is RM50. This means that
a single contract would be worth in excess of RM100,000 if the FBM KLCI is
above 2,000 points.

Since the exposures are huge, there is very few retail or individual players in SIF
contracts. The main players would therefore be institutions like pension funds,
insurance companies, fund management, merchant banks, asset management
companies, mutual funds and so on.

4.5.3 Pricing of Stock Index Futures (SIF) Contracts


Cost of carry (COC) model for futures contract is:

Ft, T = S0 (1 + rf + c ă y)t, T

Where:

rf = Risk free interest rate

Ft, T = Futures prices of SIF, maturity t to T


S0 = Spot price of the underlying stock index

y = Yield if any from holding the underlying asset

c = Storage and handling costs

We have to take note that, unlike the commodity futures contract, two
adjustments to COC model would be required:

(a) The storage/handling cost of financial futures contracts like SIF is


practically zero. Thus, the variable ‰c‰ drops off from the equation.

(b) Since holding a portfolio of stocks would enable one to receive any
dividends declared, the variable y can be replaced with d, to denote the
dividend yield. Therefore, the new formula will be:

Ft, T = S0 (1 + rf ă d)t, T
46  TOPIC 4 HEDGING STRATEGIES USING FUTURES

Let us look at Example 4.2.

Example 4.2:
Assume the following:

(a) The spot index, the FBM KLCI is now 1,700 points;

(b) The average annual dividend yield of the FBM KLCI is 1.5%;

(c) The risk free interest rate is 5% annualised; and

(d) Index multiplier is RM50.

What would be the correct price of a SIF contract if it matures in:

(a) Three months;

(b) Six months; and

(c) One year?

Solution:
The calculations for these three phases are shown in Table 4.6.

Table 4.6: The Calculations for the Correct Price of a SIF Contract

Three Months Six Months One Year


Ft, T = S0 (1 + rf ă d)t, T Ft, T = S0 (1 + rf ă d)t, T Ft, T = S0 (1 + rf ă d)t, T

F3= 1,700 (1 + 0.05 ă F6= 1,700 (1 + 0.05 ă F1yr = 1,700 (1 + 0.05 ă


0.015)1/4 0.015)1/2 0.015)1
= 1,714.68 = 1,729.49 = 1,759.50
The index multiplier is RM The index multiplier is The index multiplier is
50; thus 1,714.68  RM50, RM50; thus 1,729.49  RM50 RM50; thus 1,759.50  RM50
= RM85,734.00 = RM86,474.50 = RM87,975.00
TOPIC 4 HEDGING STRATEGIES USING FUTURES  47

4.5.4 Application of Stock Index Futures (SIF)


Contracts
We can apply SIF contracts in the following situations:

(a) Index Arbitrage


What does index arbitrage stand for?

Index arbitrage is the process of arbitraging between SIF and the spot
market.

Arbitrage is possible whenever the futures-spot parity is violated. What are


the strategies for index arbitrage? Let us look at Table 4.7 for the answer.

Table 4.7: Strategies of Index Arbitrage

Situations Strategy
If ft > S0 (1 + rf ă d)t, then the futures Short the futures contract and long the
is overpriced relative to spot or spot market.
equivalently, the quoted futures price
is higher than what it should be.
Put differently, buy from spot market
and sell the futures contract.
If ft < S0 (1 + rf ă d)t, then the futures Long the futures contract and short the
is underpriced relative to spot or the spot market.
quoted futures price is lower than
what it should be.

Let us look at an example.

Example 4.3:
Suppose the following information is given:

(i) Three months SIF price = 1,720

(ii) Index value = 1,700 points

(iii) rf rate = 5%

(iv) Dividend yield = 1.5%

(v) Time to maturity of SIF = 90 days


48  TOPIC 4 HEDGING STRATEGIES USING FUTURES

Find the correct value of the three months SIF.

Solution:
The correct value of the three months SIF should be:
Ft, T = S0 (1 + rf ă d)t, T
F90 = 1,700 (1 + 0.05 ă 0.015)1/4
= 1,714.68

This shows the futures is clearly overpriced relative to spot. The futures
price should be 1,714.68 points, yet it is quoted at 1,720 points and is
overpriced by approximately 5 points.

(b) Hedging
In the case of hedging, we are going to use the same example as illustrated
in Example 4.1 to explain the hedging process. The information given in
Example 4.1 is as follows:

(i) Current value of portfolio = RM2,125,000;

(ii) rf rate = 5% per year;


(iii) Dividend yield on portfolio = 1.5% annualised;

(iv) Spot index value = 1,700 points;

(v) Six months SIF futures contract = 1,715.00 points;

(vi) Beta of portfolio = 1.20; and

(vii) Assume the futures will expire in exactly 180 days.

Let us use the information in the following scenarios (refer to Table 4.8 and
Table 4.9).
TOPIC 4 HEDGING STRATEGIES USING FUTURES  49

Table 4.8: Scenario 1: The Stock Market Rises 10%

Action Position Today Position at Maturity Profit/Loss


a Portfolio value 2,125,000 2,380,000* 255,000
b Short 30 SIF 30  1,715  50 2,125,000  1.1  1.2
contracts 232,500
= 2,572,500 = 2,805,000
c Dividends
received till - - 15,937.50**
maturity
Net 38,437.50
Note:
* Since the beta is 1.2, if the market is up 10%, the value of portfolio is up by
12%. Therefore, 2,125,000  (1 + 0.12) = 2,380,000
** Since the annual dividend is 1.5%, 2,125,000  0.15/2= 15,937.50

Table 4.9: Scenario 2: The Stock Market Falls 10%

Action Position Today Position at Maturity Profit/Loss


a Portfolio value 2,125,000 1,870,000* 255,000
b Short 30 SIF 30  1,715  50 2,125,000  0.9  1.2
contracts 277,500
= 2,572,500 = 2,295,000
c Dividends
received till - - 15,937.50**
maturity
Net 38,437.50

Note:
* Since the beta is 1.2, if the market is up 10%, the value of portfolio is down by
12%. Therefore, 2,125,000  (1 ă 0.12) = 1,870,000.
** Since the annual dividend is 1.5%, 2,125,000  0.15/2= 15,937.50
50  TOPIC 4 HEDGING STRATEGIES USING FUTURES

Finally, Table 4.10 summarises the analysis of the hedge position.

Table 4.10: The Analysis of Hedge Position

Scenario 1 Scenario 2
(a) Initial value of portfolio 2,125,000 2,125,000
(b) Unhedged portfolio value 2,380,000 1,870,000
(d) Profit/loss from SIF contracts 232,500 277,500
(e) Dividends received 15,937.5 15,937.5
(f) Value of portfolio with hedge 2,163,437.5 2,163,437.5
(e =b + c + d)
Growth of portfolio by: 38,437.5 38,437.5

4.6 SINGLE STOCK FUTURES (SSF)


Before we end this topic, let us look at single stock futures (SSF). Firstly, what
does it mean?

Single stock futures (SSF) are a futures contract on an individual listed stock.

Being an equity futures contract, SSFs share many common features with stock
index futures (SIF) contracts. SSFs can be used for the three key applications
namely hedging, arbitrage and speculation. Like other financial derivatives, SSF
contracts are cash settled at maturity.

In Malaysia, SSF were introduced in Bursa Malaysia in April 2006. Bursa


Malaysia has 10 SSFs as listed in Figure 4.2.
TOPIC 4 HEDGING STRATEGIES USING FUTURES  51

Figure 4.2: Ten SSFs listed in Bursa Malaysia

4.6.1 Contract Specifications of Single Stock Futures


(SSF)
Now let us look at contract specifications of SSF as stated in Figure 4.3.
52  TOPIC 4 HEDGING STRATEGIES USING FUTURES

Figure 4.3: Contract specifications of single stock futures


Source: http://www.opf.com.my/futures-products/futures-exchanges/bursa-malaysia-
derivatives-bmd/contract-specifications-for-single-stocks-futures-ssfs/
TOPIC 4 HEDGING STRATEGIES USING FUTURES  53

Operationally, trading SSF contracts is very similar to that of SIF contracts. Why
do we use SSF contracts? As a derivative instrument, SSF:

(a) Requires only initial margins ă Provides automotive leverage;

(b) Has lower transaction costs ă Can be used to lower risk (hedging) to short a
stock; and

(c) Can be used to alter the beta of a portfolio.

Let us look at Illustration 4.1 on trading SSF contracts.

Illustration 4.1:
Mr Burhan is optimistic about the telecommunication industry and intends to
participate in a potential rally. He thinks Telekom Malaysia would be a big
beneficiary in the telecommunication industry. It is now June 2016; he therefore
goes long (buys) 1 September 2016 SSF contract on Telekom Malaysia at RM10.
What this means is that he gets to ‰buy‰ 2,000 Telekom Malaysia stocks on the
maturity day of the SSF contract at RM10 each or RM20,000 for the 2,000 stocks.

On the day he initiates the contract, Mr Burhan will have to place an initial
margin. Depending on his futures broker, this can vary between 10 to 25 per cent.
Variation margins also apply. Depending on which way the underlying Telekom
MalaysiaÊs stock performs, Mr Burhan may receive margin calls if the stockÊs
price declines subsequently. On the other hand, his margin account will increase
if Telekom MalaysiaÊs stock rises.

Last but not least, let us look at other applications of SSF which are:

(a) To hedge or take advantage of firm-specific events.

(b) SSF contracts are often used by fund managers to lock-in a target sell price
on a stock.

(c) SSF can be used to temporarily alter the beta of a portfolio. With SSF
contracts, a fund manager alters the overall portfolio beta by adjusting the
beta of a single stock within the portfolio.

(d) SSF can be used for executing paired-trading. Paired-trading is essentially


taking simultaneous positions in two stocks. However, the positions are
opposite to each other, that is, a long position in one and a short position in
the other.
54  TOPIC 4 HEDGING STRATEGIES USING FUTURES

SELF-CHECK 4.3

Differentiate between stock index futures (SIF) and single stock


futures (SSF) with one example each.

 The basic question of hedging is how to take a position that reduces or


neutralises the risk as much as possible.

 A long futures hedge is applicable when you buy a financial asset in the
future and are thinking of locking in the price today.

 A short futures hedge is more appropriate when you will sell a financial
asset in the future and are thinking of locking in the price today.

 According to Hull (2012), the arguments in favour of hedging are readily


apparent while the arguments against hedging are somewhat more
understated.

 Basis risk arises because of the uncertainty about the basis when the hedge is
closed out.

 We can find out the hedge ratio by dividing the size of the exposure by size
of the position taken in the futures contracts. In other words,

Size of the exposure


Hedge ratio 
Size of the position taken in the futures contracts

 A stock index futures (SIF) contract is an exchange traded futures contract


which has its underlying asset in a basket of common stocks. The basket of
common stocks would together make up an index. SIF is used for index
arbitrage and hedging.

 Single stock futures (SSF) are a futures contract on an individual listed stock.
They can be used for the three key applications namely hedging, arbitrage
and speculation. Like other financial derivatives, SSF contracts are cash
settled at maturity.
TOPIC 4 HEDGING STRATEGIES USING FUTURES  55

Application of stock index futures (SIF) Minimum variance hedge ratio


contracts
Pricing of stock index futures (SIF)
Basis risk contracts

Contract specifications of single stock Single stock futures (SSF)


futures (SSF)
Stock index futures (SIF) contract
Hedging
Trading of single stock futures (SSF)
Index arbitrage

Main players in stock index futures


(SIF) contracts

Bacha, O. I. (2012). Financial derivatives: Markets and applications in Malaysia


(3rd ed.). Shah Alam, Malaysia: McGraw-Hill.

Hull, J. C. (2012). Options, futures, and other derivatives (8th ed.). Boston, MA:
Prentice Hall.
Topic  Swaps
5
LEARNING OUTCOMES
By the end of the topic, you should be able to:
1. Identify basic principles of interest rate swaps (IRS);
2. Explain the applications of interest rate swaps (IRS);
3. Determine the value of the interest rate swaps and currency swaps;
and
4. Consider the risks of interest rate and currency swaps.

 INTRODUCTION
As another form of derivative markets, the swap is also an important component.
There are specific swaps on different financial assets.

However, in this topic, we are going to explain swaps on interest rate and
currency only. At the same time, we will examine the mechanics of interest and
currency swaps and their valuations.

5.1 INTEREST RATE SWAPS (IRS)


What do swaps mean?

Swaps are customised bilateral transactions in which the parties agree to


exchange cash flows at fixed periodic intervals, based on an underlying asset.
Bacha & Mirakhor (2013)
TOPIC 5 SWAPS  57

How about interest rate swap (IRS)? What does it stand for?

An interest rate swap (IRS) is a transaction in which the parties exchange


cash flows based on two different interest rates.

The most common form of IRS is a fixed-for-floating swap. In this type of swap,
one party pays an amount based on a fixed interest rate whereas the other party
pays in exchange, an amount based on a floating interest rate.

The size of the payment is determined by multiplying the interest rate with the
notional principal (Bacha & Mirakhor, 2013). What does notional principal mean?

The notional principal is the principal amount on which interest payments


are calculated.

Take note that the notional principal remains unchanged over the maturity of the
swap. In an IRS, cash flows are swapped at fixed predetermined intervals over
the tenor of the agreement. The fixed intervals, known as reset periods may be
semi-annually, quarterly and so on. Let us look at an illustration on fixed-for-
floating.

Illustration 5.1:
The following Figure 5.1 outlines the cash flows involved in a fixed-for-floating
IRS of 3 year maturity and RM50 million notional principal. If the reset frequency
is six months, then a total of six cash flow swaps will occur over the three years.

Figure 5.1: Cash flows involved in a fixed-for-floating IRS of three years maturity and
RM50 million notional principal

Let us look at two scenarios based on Figure 5.1.


58  TOPIC 5 SWAPS

Scenario 1:
Six months KLIBOR (Kuala Lumpur Interbank Offer Rate)= 5%
The payment obligation for each party is as follows:

Fixed rate payer: [8% ï RM50,000,000]/2 = RM2,000,000


Floating rate payer: [(5% + 1%)  RM50,000,000]/2 = RM1,500,000

Since the fixed rate payerÊs obligation is higher by RM500,000, he pays this
amount. The netted cash flow will be (see Figure 5.2):

Figure 5.2: The netted cash flow for Scenario 1

Scenario 2:
Six months KLIBOR = 10%
The payment obligation for each party is as follows:

Fixed rate payer: [8%  RM50,000,000]/2 = RM2,000,000


Floating rate payer: [(10% + 1%)  RM50,000,000]/2 = RM2,750,000

Since the floating rate payerÊs obligation is higher, he has to pay the fixed rate
pay the net amount of RM750,000. Now the cash flow will be (see Figure 5.3):

Figure 5.3: The cash flow for Scenario 2

There are certain terminologies of swaps that you need to know. These
terminologies are listed in Table 5.1.
TOPIC 5 SWAPS  59

Table 5.1: Terminologies of the Swaps

Terminology Meaning
Fixed rate payer The party in swaps contracts who pays based on a fixed interest rate.
Floating rate The other party who pays based on a floating interest rate.
payer
Reset frequency The time interval over which the floating rate is reviewed and reset.
Reference rate The market interest rate on which the floating rate payerÊs payment
will be based ă like KLIBOR, LIBOR (London Interbank Offer Rate),
T-bill (Treasury bill) rate and so on.
Notional The principal amount on which interest payment amounts are
principal determined. Notional amount is never exchanged, only the interest
amounts based on it.

5.1.1 Why Use Interest Rate Swaps (IRS)?


IRS grew out of parallel or back-to-back loans and currency swaps that
multinational firms had used in the 1970s to access blocked funds or circumvent
capital controls. Since its introduction in 1981, IRS has become extremely
popular.

Other variants of fixed-for-floating IRS are:

(a) A basis swap is one where both rates are floating and parties try to lock-in
or profit from differences in swap spreads;

(b) A forward-starting swap enables parties to lock-in current favourable


yields for financing issues that will be carried out at a future date;

(c) An amortising swap is one where the notional principal reduces over time;
and

(d) A swap with an inverse floater is one where the floating rate is negatively
correlated with a benchmark reference interest rate.

SELF-CHECK 5.1

1. Explain the basic principles of interest rate swaps (IRS).

2. State other variants of fixed-for-floating IRS.


60  TOPIC 5 SWAPS

5.2 APPLICATIONS OF INTEREST RATE SWAPS


(IRS)
Did you know that IRS can also be used to lower the cost of borrowing and
manage asset-liability duration mismatches, aside from their use in hedging,
arbitrage and speculation? Let us learn more on these applications in the next
sections.

5.2.1 Arbitrage Using IRS


Did you know that one of the main usages of IRS was as an instrument to take
benefit of comparative advantage which companies may have across different
credit markets? This is because different markets price credit differently based on
their own assessment. It is this mispricing that can be arbitraged (Bacha, 2012;
Hull 2012). Let us look at Illustration 5.2 to understand more on arbitrage.

Illustration 5.2:
Suppose two companies, Wangsa and Melati are in the market for a RM20
million, three years loan. Principal is to be paid in a single payment at the end of
year 3 with semi-annual interest payments in between. Wangsa which has a
„AAA‰ credit rating prefers a floating rate loan, whereas Melati which has an
‰A‰ rating wants a fixed rate loan. The rates available to each company in the
fixed and floating markets are as follows (refer to Table 5.2):

Table 5.2: The Rates Available to Each Company in the Fixed and Floating Markets

Company Fixed Rate Floating Rate


Wangsa 6% KLIBOR + 0.25%
Melati 7% KLIBOR + 0.75%

The 0.5% difference that spreads between the two markets can be thought of as
mispricing. This illustration can be simplified into Figure 5.4.
TOPIC 5 SWAPS  61

Figure 5.4: Illustration of interest rate swaps

Wangsa has an absolute advantage in both markets vis-à-vis Melati (its cost is
lower). However, it has a bigger comparative advantage in the fixed rate market.

In structuring the swaps, Wangsa should be made to borrow in the market in


which it has the bigger comparative advantage. However, Wangsa wants a
floating rate loan. This is where it swaps its fixed rate loan into a floating rate.
With the IRS, there are two things happen:

(a) Each company effectively ends up with the kind of loan it originally
wanted; and

(b) They manage to get their preferred loan at a lower cost than if they had
borrowed directly.

This can be simplified into Table 5.3.

Table 5.3: Swaps Structuring for Wangsa and Melati

Activity Wangsa Melati


Cost of borrowing directly KLIBOR + 0.25% 7%
Cost of borrowing with IRS KLIBOR 6.875%
Cost saving from IRS +0.25% +0.125%
62  TOPIC 5 SWAPS

Where is the rest of 0.125% out of 0.5%? It is considered as gain to HSBC (see
Table 5.4).

Table 5.4: Summary for Illustration 5.2

Activity Rate
Receive from Melati 6.875%
Pay to Wangsa 6%
Receive from Wangsa KLIBOR
Pay to Melati (KLIBOR + 0.75%)
Gain to HSBC 0.125%

5.2.2 Hedging Using IRS


In hedging, IRS can be used to hedge against rising rates, falling rates and
manage asset-liability duration gaps. Let us look at rising rates and falling rates
illustrations.

(a) Illustration 5.3: Hedging Rising Interest Rates


Suppose a borrower, Company Maju, has a 3 year, RM5 million loan from
Public Bank. Public Bank charges an interest based on 6 month KLIBOR +
1% payable semi-annually. The funding costs of the company will increase
if the 6 month KLIBOR rises. How can the firm hedge?

The logical way for Company Maju to manage the rate risk would be
through an IRS (see Figure 5.5).

Figure 5.5: Illustration of hedging process by using interest rates rising


TOPIC 5 SWAPS  63

In the IRS, Company Maju pays an annualised fixed rate of 6% every six
months on RM5 million notional [(0.06  RM5 million)/2].

In exchange, it receives a cash flow equivalent to the prevailing spot six


months KLIBOR rate on RM5 million notional [(6 months KLIBOR  RM5
million)/2].

Company Maju protects itself from rising interest rates because its
increased payments to Public Bank will be offset by the increased payments
it receives from the counterparty as interest rates rising. With the IRS,
Company Maju has effectively turned its floating rate payable into a fixed
rate one.

(b) Illustration 5.4: Hedging Rising Interest Falling


One of the money brokers manages a money-market fund for corporate
clients. The size of the fund is RM100 million, invested mostly in six months
Malaysian government securities and commercial papers. The returns
approximate the six months KLIBOR.

Based on its in-house research, this money broker (MB) company expects
short-term rates to fall over the next few years. Therefore, the company is
worried that falling short-term rates will mean lower yields from its
investments and reduced returns to its clients.

What can this MB company do to maintain steady returns in the face of


falling short term rates?

Enter an IRS of RM100 million notional principal as the floating rate payer.
In the swap, MB essentially passes through its earnings from six months
paper to the counterparty as the six months KLIBOR. The MB Company
receives in exchange a fixed rate of X%. As a result, MB Company can be
assured of providing its clients with a return approximating X% even
though short term rates are falling. This solution can be simplified into
Figure 5.6.
64  TOPIC 5 SWAPS

Figure 5.6: Illustration of hedging process by using interest rates falling

SELF-CHECK 5.2

Describe the applications of interest rate swaps (IRS).

5.3 VALUATION OF INTEREST RATE SWAPS


(IRS)
Did you know that pricing interest rate swaps is a relatively complicated process
requiring extrapolation and estimation? There is no formula providing a closed
form solution. Instead, the process requires sourcing information from several
sources, then estimating value using techniques such as bootstrapping and
extrapolation.

What is the underlying logic of valuing a swap? The underlying logic of valuing
a swap is based on the premise that the transaction should (at its initiation) be a
zero net present value (NPV) proposition.

Take note that a number of variables have to be estimated. First, to compute


present values (PVs), the appropriate discount rate has to be estimated. This is
done by estimating the swap spread. What does swap spread mean?

The swap spread is essentially the premium above the treasury yield
applicable for the swap.
TOPIC 5 SWAPS  65

Since the spread is a premium over treasury bills, a swap curve is often estimated
based on the treasury yield curve.

The second task is to estimate the potential cash flow from the floating rate
payer. Since this is dependent on the movements of the floating reference rate, a
forecast has to be made of what the reference rate is likely to be in future periods.
In highly liquid and deep markets, the reference rate is forecasted by looking at
the forward yield curve. What does forward yield curve mean?

The forward yield curve is typically derived from pricing of forward based
interest rate derivatives such as interest rate futures contracts.

Thirdly, once the expected future floating rate is estimated, the cash flow is
determined by multiplying the estimated yield with the notional principal. This
estimated cash flow is then discounted and set equal to the discounted cash flow
arising from the fixed rate payment.

The stated valuation method sets the value of an IRS to be net zero NPV at
initiation. However, as market interest rates change, the value of the IRS would
change too.

5.4 CURRENCY SWAPS


Firstly, what is a currency swap?

A currency swap is a transaction in which two parties agree to exchange a


fixed amount of one currency for another.

The rationale for such a transaction is to hedge currency risk and the interest rate
risk that typically goes with it. The main difference between IRS and a currency
swap is that in an IRS, notional principal is never exchanged, whereas in a
currency swap, the notional principal in two different currencies is exchanged.
Basically, this first exchange is often based on prevailing spot exchange rates at
the time. Let us look at an illustration that demonstrates this matter.

Illustration 5.5:
Top Glove (TG) is a Malaysian glove and rubber products manufacturer which
has a large and burgeoning market in China. The company feels that it has to
have its own warehouse and distribution centre in China. The total cost for land,
66  TOPIC 5 SWAPS

building and all shipping and handling equipment is expected to be RMB100


million.

This will be a one-off investment with subsequent capital expenditure expected


to be minimal. Given its expected cash flows from China operations, TG believes
it can settle a RMB100 million financing in three years. Subsequently, the
company has negotiated with its Chinese banker, The Agricultural Bank of China
(ABC), for funding as follows:

(a) Principle amount = RMB100 million;

(b) Interest = Fixed 7% payable annually at year end; and

(c) Loan tenor = Three years; with lump sum principle payment at end of 3rd
year.

Saba, the Chinese food industry has operations in Malaysian Palm Oil industry.
It now wants to build its own refining facilities in Shah Alam, Malaysia. The
total investment needed will be RM50 million. SabaÊs Malaysian banker,
Maybank is willing to provide funding as follows:

(a) Principle amount = RM50 million;

(b) Interest = Fixed 5% payable annually at year end; and

(c) Loan tenor = Three years; with lump sum principle payment at end of 3rd
year.

Since the two companies have opposite needs, a currency swap can be a means
by which both companies manage the exchange rate risk.

If each company takes the loan being offered by its bank without doing anything
more, they face exchange rate risk on both the principal amount and the annual
interest payments. To see how the swap can be structured, assume that the spot
exchange rate between the RMB and the ringgit is RMB2 per ringgit. This is to
lock-in the prevailing exchange rate and avoid any currency risk on both the
principal and interest. The currency swap will work as follows:

(a) TG takes the loan principal of RMB100 million from ABC and forwards it to
Saba;

(b) In turn, Saba gives to TG the RM50 million it has received from Maybank;
and
TOPIC 5 SWAPS  67

(c) TG now takes the RM50 million received from Saba, converts it at the spot
rate to RMB100 million. These principal amounts are then reversed at the
end of 3rd year.

This transaction can be simplified into Figure 5.7.

Figure 5.7: Illustration of currency swaps

SELF-CHECK 5.3

1. Explain the valuation of interest rate swaps (IRS).

2. Describe currency swaps with one appropriate example.

5.5 RISKS OF INTEREST RATE AND CURRENCY


SWAPS
Finally, this subtopic will discuss the risks associated with interest rate and
currency swaps. Hopefully by the end of this subtopic, you will able to identify
these risks (see Table 5.5).
68  TOPIC 5 SWAPS

Table 5.5: Risks Associated with Interest Rate and Currency Swaps

Risk Description
Interest rate Interest rates might move against the swap bank after it has only
risk gotten half of a swap on the books, or if it has an unhedged position.
Basis risk If the floating rates of the two counterparties are not pegged to the
same index.
Exchange rate In the example of a currency swap given earlier (Illustration 5.5), the
risk swap bank would be worse off if the pound appreciated.
Credit risk This is the major risk faced by a swap dealer ă the risk that a counter
party will default on its end of the swaps.
Mismatch risk It is hard to find a counterparty that wants to borrow the right
amount of money for the right amount of time.
Sovereign risk The risk that a country will impose exchange rate restrictions that will
interfere with performance on the swaps.

SELF-CHECK 5.4

Describe the risks of interest rate and currency swaps.

 An interest rate swap (IRS) is a transaction in which the parties exchange cash
flows based on two different interest rates.

 The most common form of IRS is a fixed-for-floating swap.

 IRS is commonly used in arbitrage, hedging and speculation. It can also be


used to lower the cost of borrowing and manage asset-liability duration
mismatches.

 The underlying logic of valuing a swap is based on the premise that the
transaction should (at its initiation) be a zero net present value (NPV)
proposition.

 A currency swap is a transaction in which two parties agree to exchange a


fixed amount of one currency for another. The rationale for such a transaction
is to hedge currency risk and the interest rate risk that typically goes with it.
TOPIC 5 SWAPS  69

 There are six risks associated with interest rate and currency swaps:

 Interest rate risk;

 Basis risk;

 Exchange rate risk;

 Credit risk;

 Mismatch risk; and

 Sovereign risk.

Applications of interest rate swaps Interest rate swaps (IRS)


(IRS)
Mechanism of interest rate swaps (IRS)
Arbitrage using interest rate swaps
(IRS) Risks of interest rate and currency
swaps
Currency swaps
Swaps
Hedging using interest rate swaps
(IRS) Valuation of interest rate swaps (IRS)

Bacha, O. I. (2012). Financial derivatives: Markets and applications in Malaysia


(3rd ed.). Shah Alam, Malaysia: McGraw-Hill.

Bacha, O. I., & Mirakhor, A. (2013). Islamic capital markets: A comparative


approach. Singapore: John Wiley & Sons.

Hull, J. C. (2012). Options, futures, and other derivatives (8th ed.). Boston, MA:
Prentice Hall.
Topic  Mechanics of
6 Options
Markets
LEARNING OUTCOMES
By the end of the topic, you should be able to:
1. Describe options and the specifications of stock options;
2. Differentiate between call options and put options;
3. Describe the option moneyness;
4. Explain the trading, commissions and margins related to option
contracts; and
5. Summarise the options clearing corporation (OCC).

 INTRODUCTION
In this topic, we will look into another important financial derivative instrument,
namely, options. What are options? In the forward and futures contracts, the
holders of the contracts have to fulfil their promises. Put differently, both parties
are legally bound by the contract. However, options provide the holder the right
but not the obligation to exercise.

What else do you need to know about options? In this topic, you will be
introduced to specification of stock options, two types of options (call options
and put options) and option moneyness. Then, your learning continues on
trading, commissions and margins related to option contracts as well as the
options clearing corporation (OCC). Are you ready to discover more? Let us
continue the lesson.
TOPIC 6 MECHANICS OF OPTIONS MARKETS  71

6.1 OPTIONS
Historically, the exchange traded options were introduced in 1973 on The
Chicago Board Options Exchange (CBOE). The initial option contracts were
written on equities.

Currently, a wide variety of underlying instruments are used for options. For
examples, stock indexes, foreign currencies, commodities and other derivatives
(interest rate futures, stock index futures or swaps).

There are two basic forms of options namely call options and put options (refer
to Table 6.1).

Table 6.1: Two Forms of Option Contracts

Call Options Put Options


Call options provide the holder the right Put options provide the holder the right
but not the obligation to buy the but not the obligation to sell the underlying
underlying asset at the predetermined asset at the predetermined exercise price.
exercise price.

In the case of the predetermined price, the price at which the transaction will be
carried is known as the exercise price or strike price. In addition, option contracts
are classified into two forms according to exercising rights (refer to Table 6.2).

Table 6.2: Two Exercising Styles of Option Contracts

Style Description
A European style option This option can be exercised only at maturity.
An American style option This option can be exercised at or any time before
maturity.

With this additional flexibility, an American option would be more valuable than
a European option assuming all other features are the same.

To sum up, an option contract at the very least specifies the following five
features (see Figure 6.1).
72  TOPIC 6 MECHANICS OF OPTIONS MARKETS

Figure 6.1: Five features specified by an option contract

The long position (buyer) has a right but not the obligation to exercise, whereas
the seller or short position is obliged to fulfil the buyerÊs wants should he choose
to exercise.

If the holder of the put option chooses to exercise at the exercise price, the seller
of the put option must stand ready to buy the underlying asset at the exercise
price. Table 6.3 summarises the definitions for call and put options on buyer and
seller.

Table 6.3: Definitions of Call and Put Options

Position Calls Puts


Buyer (long) Has right but not obligation to buy Has right but not obligation to
underlying asset at exercise price. sell underlying asset at
exercise price.
Seller (short) Is obliged to sell the underlying asset Is obliged to buy the
at exercise price. underlying asset at exercise
price.

6.2 SPECIFICATIONS OF STOCK OPTIONS


In this subtopic, we are going to use FTSE Bursa Malaysia KLCI Index Options
(OKLI) to explain the contract specifications of option contracts (refer to
Table 6.4).
TOPIC 6 MECHANICS OF OPTIONS MARKETS  73

Table 6.4: Specifications of Option Contracts

FTSE Bursa Malaysia KLCI Index Options (OKLI) Specifications

EXCHANGE Bursa Malaysia Derivatives (BMD)


UNDERLYING FBM KLCI Futures (FKLI)
INSTRUMENT
TYPE European Style
CONTRACT SIZE One FKLI contract
TICK SIZE 0.1 index point valued at RM5
CONTRACT Spot month, the next month, and next two calendar quarterly
MONTHS months. The calendar quarterly months are March, June,
September and December
TRADING HOURS First trading session: Malaysian 8:45am to 12:45pm
Second trading session: Malaysian 2:30pm to 5:15pm
LAST TRADING The last business day of the contract month.
DAY
EXERCISE PRICE At least 13 exercise prices (six are in-the-money, one is at-the-
INTERVAL money and six are out-of-the-money) shall be set at interval of
10 index points for the spot and next month contracts. At least
seven exercise prices (three are in-the-money, one is at-the-
money and three are out-of-the-money) shall be set at interval
of 20 index points for the next two quarterly month contracts.
SETTLEMENT OF In the absence of contrary instructions delivered to the Clearing
OPTION House, an option that is in-the money at expiration shall be
EXERCISE automatically exercised. Exercise results in a long FKLI
position, which corresponds with the optionÊs contract month
for a call buyer or a put seller and a short FKLI position for a
put buyer or a call seller. The resultant positions in FKLI shall
then be cash-settled based on the final settlement value of FKLI.
SPECULATIVE 10,000 FKLI-equivalent contracts (a combination of OKLI and
POSITION LIMIT FKLI contract), net on the same side of the market in all
contract months combined.

Source: Oriental Pacific Futures Sdn Bhd


74  TOPIC 6 MECHANICS OF OPTIONS MARKETS

Next are some terminologies of option contracts as stated in Table 6.5.

Table 6.5: Terminologies of Option Contracts

Terminology Description
Expiry  The last business day of the contract month before 5:15pm.
Strike price  When a new expiration date is introduced, the two or three strike
prices closest to the current stock price are usually selected by the
exchange.
 If the stock prices move outside this range, new options will be
introduced.
Dividends and  Exchange traded options are adjusted for stock splits and stock
splits dividends.
 Exchange traded options are generally not adjusted for cash
dividends.
Position limits  A position limit is the maximum number of option contracts that
an investor can hold on one side of the market.
 Short calls and long puts are on the same side.
Exercise limits  Equals the position limit.

SELF-CHECK 6.1

1. Define options.

2. Describe the two forms of option contracts.

3. Explain the specifications of stock options.

4. State the terminologies of option contracts.

6.3 CALL OPTIONS


As mentioned earlier, call options provide the holder the right but not the
obligation to buy the underlying asset at the predetermined exercise price.

Basically, there are two parties in any kind of contract, that is, the buyer and
seller. In this case of call option, the buyer of call options is called long call
whereas the seller of call options is called short call.
TOPIC 6 MECHANICS OF OPTIONS MARKETS  75

6.3.1 Long Call


Long call is the buyer of call options. In this case, the terminal stock price is more
than exercise price, then the buyer of call options will profit from the investment.
For example, profit from buying a European call option on stock ABC is as
follows:

(a) Option price = RM0.5; and

(b) Strike price = RM20.

This example can be graphed as in Figure 6.2.

Figure 6.2: Long call

6.3.2 Short Call


Basically, the short call is the seller of call options. Let us say profit from selling a
European call option on stock ABC is as follows:

(a) Option price = RM0.5; and

(b) Strike price = RM20.

This example can be graphed as in Figure 6.3.

Figure 6.3: Short call


76  TOPIC 6 MECHANICS OF OPTIONS MARKETS

The breakeven point for calls is the exercise price plus premium. When we
combine these two positions, we get a new graph as shown in Figure 6.4.

Figure 6.4: Combination of long and short calls

Based on the graph in Figure 6.4, we can see that the long position in call options
will gain if the price of the underlying asset is up. In this case, the profit of long
call is technically unlimited, but the loss is limited. The short position in call
options only gains if the price of underlying asset is down. However, the short
call only gains the premium paid, the loss is unlimited if the price of the
underlying asset is up.

SELF-CHECK 6.2

Explain both long and short call options with the help of a diagram.

6.4 PUT OPTIONS


In the case of put options, it provides the holder the right but not the obligation
to sell the underlying asset at the predetermined exercise price. As contractors,
the buyer of put option is called long put whereas the seller of put options is
called short put.
TOPIC 6 MECHANICS OF OPTIONS MARKETS  77

6.4.1 Long Put


In the put options, if the terminal stock price is less than exercise price, then the
buyer of put option will profit from the investment. Let us say that profit from
buying a European put option on stock DEF is as follows:

(a) Option price = RM0.5; and

(b) Strike price = RM20.

This long put option can be graphed as in Figure 6.5.

Figure 6.5: Long put

6.4.2 Short Put


Let us say that profit from selling a European put option on stock DEF is as
follows:

(a) Option price = RM0.5; and

(b) Strike price = RM20.

This profit can be visualised into graph (see Figure 6.6).

Figure 6.6: Short put


78  TOPIC 6 MECHANICS OF OPTIONS MARKETS

The breakeven point for puts is the exercise price minus premium. When we
combine these two positions, we get a new graph as shown in Figure 6.7.

Figure 6.7: Combinations of long and short puts

Based on the graph in Figure 6.7, we can see that the long position in put options
will gain if the price of the underlying asset is down. In this case, the profit of
long put is technically unlimited, but the loss is limited. The short position in put
options only gains if the price of the underlying asset is up. However, the short
put only gains the premium paid, the loss is unlimited if the price of underlying
asset is down.

SELF-CHECK 6.3

Explain both long and short put options with the help of a diagram.

ACTIVITY 6.1

An investor buys a call option with a strike price of RM45 and a put
option with a strike price of RM40. Both options have the same
maturity. The call costs RM3 and the put costs RM4. Draw a diagram
that shows the variation of the traderÊs profit with the asset price.
TOPIC 6 MECHANICS OF OPTIONS MARKETS  79

6.5 OPTION MONEYNESS


Firstly, what does option moneyness refer to?

Option moneyness refers to terminologies used to describe whether an


option is currently profitable.

There are three different types of option moneyness as stated in Figure 6.8.

Figure 6.8: Three different types of option moneyness

These three are further elaborated as follows:

(a) In-the-Money (ITM)


When is an option said to be in-the-money? An option is said to be in-the-
money if exercising it now will mean profit. For a call to be in-the-money,
its exercise price must be lower than the current value or spot price of the
underlying asset. For a put option to be in-the-money, its exercise price
must be higher than the value of the underlying asset. To simplify:

(i) Call: Exercise price < Spot price of underlying

(ii) Put: Exercise price > Spot price of underlying

(b) At-the-Money (ATM)


An option is said to be at-the-money if its exercise price equals the spot
price of the underlying or simply:

(i) Call: Exercise price = Spot price of underlying

(ii) Put: Exercise price = Spot price of underlying

(c) Out-of-the-Money (OTM)


An option is said to be out-of-the-money if exercising it will mean losses. A
call would be out-of-the-money if its exercise price is higher than the
80  TOPIC 6 MECHANICS OF OPTIONS MARKETS

current spot price of the underlying asset. A put would be out-of-the-


money if its exercise price is lower than the underlying assetÊs spot price.
This can be simplified as:

(i) Call: Exercise price > Spot price of underlying

(ii) Put: Exercise price < Spot price of underlying

SELF-CHECK 6.4

1. Define option moneyness.

2. Describe three different types of option moneyness.

6.6 TRADING, COMMISSIONS AND MARGINS


This subtopic will discuss trading, commissions and margins which are related to
option contracts (refer to Table 6.6).

Table 6.6: Description of Trading, Commissions and Margins

Elements Description
Market Makers
 Offer bid and ask quotes on the option.
 Market makers ensure that buying and selling orders can always be
executed at some price without delay.
 The bid-ask spread is their compensation for this liquidity
Trading intermediation.
Offsetting Orders
 An investor can close out a long position by issuing an offsetting
order to sell the same option.
 An investor can close out a short position by issuing an offsetting
order to buy the same option.
Commissions  Commissions vary substantially from one broker to another.
 Its hidden cost is the bid-ask spread.
Margins  Since options already have plenty of leverage, option premium must
be paid in full.
TOPIC 6 MECHANICS OF OPTIONS MARKETS  81

6.7 THE OPTIONS CLEARING CORPORATION


(OCC)
Lastly, let us look at the options clearing corporation (OCC). Keep in mind that
the options clearing corporation (OCC) has the same function as the clearing
house in the futures markets. This corporation guarantees that option writers
fulfil their obligations and keep a record of all long and short positions.

In OCC, exercising an option will be done as follows:

(a) The OCC randomly selects a member with an outstanding short position in
the option;

(b) The member then selects a particular investor who has written the option;
and

(c) At expiry, all in-the-money options should be exercised unless transactions


costs are too high.

 Options provide the holder the right but not the obligation to exercise.

 There are two basic forms of options, namely, call options and put options.
Each of these options has another two forms namely long (buyer) and short
(seller).

 Some of the contract specifications of option contracts are type, contract size,
tick size, contract months, trading hours and so on.

 Call options provide the holder the right but not the obligation to buy the
underlying asset at the predetermined exercise price. In the case of call
options, the buyer is called long call whereas the seller is called short call.

 Put options provide the holder the right but not the obligation to sell the
underlying asset at the predetermined exercise price. As contractors, the
buyer of put option is called long put whereas the seller of put options is
called short put.

 Option moneyness refers to terminologies used to describe whether an option


is currently profitable.
82  TOPIC 6 MECHANICS OF OPTIONS MARKETS

 There are three different types of option moneyness namely in-the-money, at-
the-money and out-of-the-money.

 Trading of options contracts can be divided into market makers and


offsetting orders.

 Commissions of options contracts vary substantially from one broker to


another. Its hidden cost is the bid-ask spread.

 Margins of options contracts have plenty of leverage; therefore, option


premiums must be paid in full.

 The options clearing corporation (OCC) has the same function as the clearing
house in the futures markets. It guarantees that option writers fulfil their
obligations and keep a record of all long and short positions.

At-the-money Options clearing corporation (OCC)

Call options Out-of-the-money

In-the-money Put options

Long call Short call

Long put Short put

Option moneyness Stock options specifications

Options

Bacha, O. I. (2012). Financial derivatives: Markets and applications in Malaysia


(3rd ed.). Shah Alam, Malaysia: McGraw-Hill.

Hull, J. C. (2012). Options, futures, and other derivatives (8th ed.). Boston, MA:
Prentice Hall.
T op i c  Trading
7 Strategies
Involving
Options
LEARNING OUTCOMES
By the end of the topic, you should be able to:
1. Identify the four types of option strategies;
2. Describe the uncovered or naked positions;
3. Apply hedge strategies;
4. Use spread strategies;
5. Recommend combination strategies; and
6. Evaluate the strategies by market outlook.

 INTRODUCTION
Did you know that one of the main advantages of options is their flexibility? This
flexibility arises from the fact that options may or may not be exercised. This
flexibility enables an investor to establish positions that may not be possible with
other instruments. This inherent flexibility also enables options to be combined
with positions in the underlying asset and in other derivative instruments.
Therefore, an option strategy is established with an objective in mind or for a
given market outlook (Bacha, 2012).
84  TOPIC 7 TRADING STRATEGIES INVOLVING OPTIONS

The flexibility in usage of option and possible combinations with other assets
results in an infinite number of possible option strategies. For ease of clarity, the
option strategies are characterised into four broad categories as stated in Figure
7.1.

Figure 7.1: Four broad categories of options strategies

These option strategies are the main focus of this topic as well as strategies by
market outlook. Are you ready to learn more about these strategies? Let us
continue the lesson.

7.1 UNCOVERED OR NAKED POSITIONS


What does an uncovered or ‰naked‰ position mean?

An uncovered or ‰naked‰ position is where one takes a position in an asset


without establishing an offsetting position.

Using calls, put and stocks (underlying asset), there are six possible uncovered or
naked positions. These positions are listed in Figure 7.2.

Figure 7.2: Six possible uncovered or naked positions


TOPIC 7 TRADING STRATEGIES INVOLVING OPTIONS  85

SELF-CHECK 7.1

State the possible positions in uncovered or naked positions.

7.2 HEDGE STRATEGIES


A hedge strategy combines an option with the underlying asset in such a way
that the overall position either reduces or eliminates risk. A fully hedged position
is riskless. The combination is such that price movements offset each other. Three
common hedge strategies are:

(a) Portfolio insurance (hedging exposure to a long stock position);

(b) Hedging exposure to a short stock position; and

(c) Conversion strategy (locking-in a fixed value of underlying asset).

In the next subtopics, we are going to explain each of them one by one.

7.2.1 Portfolio Insurance – Long Stock Position


A long stock position exposes the investor to downward movements in the
stockÊs price. Thus, an investor with a long stock position would want to protect
downside risk while keeping as much as possible of the upside profit potential.
The investor should combine the long stock position with an option position that
would profit if the stock goes down in value.

There are two option positions gained when the underlying asset goes down in
value. They are:

(a) Short call position (receive only premium); and

(b) Long put position (will gain if the underlying asset price is down).

The short call position only provides downside protection to the extent of the
premium received while the upside potential on the stock cannot be realised by
the investor.

Therefore, the logical hedge choice would be to use the long put position which
provides upside potential if the underlying asset price is down. This strategy is
often commonly referred to as portfolio insurance. Let us look at Illustration 7.1
that demonstrates long stock position.
86  TOPIC 7 TRADING STRATEGIES INVOLVING OPTIONS

Illustration 7.1: Long Stock Position


Suppose you had just gone on a long stock position of one lot of company ABCÊs
stock at a price of RM10 each, for a total investment of RM10,000. You wish to
protect yourself from any short-term downside movement in price. Suppose
three months, at-the-money put options on company ABC stock are being quoted
at RM0.20 each.

The appropriate option strategy to hedge the long stock position would be (refer
to Table 7.1):

Table 7.1: The Appropriate Option Strategy to Hedge


the Long Stock Position for Illustration 7.1

Combined Long 1 lot company ABC stock @ RM10 Long stock


position
Long 1, three months company ABC put @ RM0.20 Long put

Payoff to hedged long stock position is shown in Table 7.2.

Table 7.2: Payoff to Hedged Long Stock Position

Stock Price Value of Long


Profit/Loss to Long Put Value of Combined
at Maturity Stock Positions
Position at RM0.20 (RM) Position at Maturity (RM)
(RM) (RM)
6 6,000 3,800 9,800
8 8,000 1,800 9,800
10 10,000 (200) 9,800
12 12,000 (200) 11,800
15 15,000 (200) 14,800

Then, the payoff profile to portfolio insurance will be as shown in Figure 7.3.
TOPIC 7 TRADING STRATEGIES INVOLVING OPTIONS  87

Figure 7.3: Payoff profile to portfolio insurance

Based on Figure 7.3, we can see the combined position; long position in a stock
and long position in a put.

To summarise, let us look at the key features of portfolio insurance (long stock
position) in Table 7.3.

Table 7.3: Key Features of Portfolio Insurance (Long Stock Position)

Strategy Description
When to use When one needs protection against falling value of the underlying
asset in which one has a long position
Risk profile Limited downside risk, unlimited upside potential
Break-even point Since overall position is that of a long call;
Exercise price + Premium = RM10.00 + RM0.20 = RM10.20
Desired objective To gain from potential upside rally while limiting downside risk

7.2.2 Hedging Exposed to a Short Stock Position


With a short stock position, the investor needs protection from a rise in the
underlying stock price. The hedge position should combine the short stock
position with an option that rises when the underlying stock price increases. The
logical strategy would be to long a call option on the stock or simply: long call
position (will gain if underlying asset price is up).
88  TOPIC 7 TRADING STRATEGIES INVOLVING OPTIONS

Let us look at Illustration 7.2 that demonstrates this.

Illustration 7.2: Short Stock Position


Suppose you shorted company DEF stock at RM20. You would like to hedge
yourself from potential large losses if the stock price rises. How can you hedge?
Let us say a three months, RM20.00 call option on the stock is being sold at a
premium of RM0.30 each.

The appropriate option strategy to hedge the short stock position would be as
shown in Table 7.4.

Table 7.4: The Appropriate Option Strategy to Hedge


the Short Stock Position for Illustration 7.2

Combined Short 1 lot of company DEF stock at RM20.00 Short stock


position
Long 1, three months RM20.00 call @ RM0.30 Long call

Then, the payoff to hedged short stock position will be as shown in Table 7.5.

Table 7.5: The Payoff to Hedged Short Stock Position

Stock Price Value of Long Value of Combined


Profit/Loss to Long Put
at Maturity Stock Positions Position at Maturity
Position at RM0.30 (RM)
(RM) (RM) (RM)
16 4,000 (300) 3,700
18 2,000 (300) 1,700
20 0 (300) (300)
22 (2,000) 1,700 (300)
25 (5,000) 4,700 (300)

The combined position for short position in a stock and long position in a call is
shown in Figure 7.4.
TOPIC 7 TRADING STRATEGIES INVOLVING OPTIONS  89

Figure 7.4: Short stock position

Lastly, the key features for hedging a short stock position are shown in Table 7.6.

Table 7.6: Key Features of Hedging a Short Stock Position

Strategy Description
When to use When one needs protection against rising values of the underlying
asset of which one is short
Risk profile Limited downside risk, unlimited upside potential
Break-even Since overall position is that of a long put;
point Exercise price ă Premium = RM20.00 ă RM0.30 = RM19.70
Desired To gain from falling prices while limiting risk associated with rising
objective prices

7.3 SPREAD STRATEGIES


What does a spread strategy mean?

A spread strategy can be thought of as a speculative position with a safety


net.
90  TOPIC 7 TRADING STRATEGIES INVOLVING OPTIONS

A spread position typically involves the establishment of offsetting positions in


the same asset but across different markets, at different maturities or different
exercise prices. Spread positions are established to profit from expected marginal
price movements while protecting downside risk.

There are two common types of spreads namely bull spreads and bear spreads.
Both these spreads can be established using either calls or puts (refer to
Table 7.7).

Table 7.7: Types of Spreads

Types of Spread Name of Strategy


Using calls  bull call spread
Bull spread
Using puts  bull put spread
Using calls  bear call spread
Bear spread
Using puts  bear put spread

7.3.1 Bull Call Spread


Let us understand more on bull call spread by looking at Illustration 7.3.

Illustration 7.3:
Suppose you are moderately bullish about the performance of Gombak BerhadÊs
stock over the next 60 days. You want to apply options to benefit from the
expected moderate stock price appreciation and at the same time you want to
minimise your downside risk.

In this case, the appropriate option strategy would be to create a bull spread
which could be set up using call options. Assume the following two call options
are available:

(a) Gombak Berhad, RM19.50 call @ RM0.40

(b) Gombak Berhad, RM20.50 call @ RM0.15

The bull call spread would require the purchase of the call with the lower
exercise price and sale of the call with the higher exercise price which is:

(a) Long RM19.50 call @ RM0.40

(b) Short RM20.50 call @ RM0.15


TOPIC 7 TRADING STRATEGIES INVOLVING OPTIONS  91

Then, a payoff of bull call spread will be calculated as shown in Table 7.8.

Table 7.8: Payoff of Bull Call Spread

Profit/Loss to long Value of


Stock Price at Profit/Loss to Short
Combined
Maturity (RM) RM19.50 Call @ RM0.40 RM20.50 Call @ RM0.15
Position
15 (0.40) 0.15 (0.25)
17 (0.40) 0.15 (0.25)
19 (0.40) 0.15 (0.25)
19.6 (0.30) 0.15 (0.15)
21 1.1 (0.35) 0.75
23 3.1 (2.35) 0.75
25 5.1 (4.35) 0.75

Lastly, we can turn Table 7.8 into a graph as shown in Figure 7.5.

Figure 7.5: Illustration of bull call spread


92  TOPIC 7 TRADING STRATEGIES INVOLVING OPTIONS

7.3.2 Bull Put Spread


A bull put spread is the use of put options instead of calls to establish the spread.
Let us refer back to Illustration 7.3. The following 60 days put options are
available:

(a) Gombak Berhad, RM19.50 put @ RM0.20

(b) Gombak Berhad, RM20.00 put @ RM0.60

The bull put spread strategy would be as follows:

(a) Long RM19.50 put @ RM0.20

(b) Short RM20.00 put @ RM0.60

Then, the payoff of bull put spread is calculated as in Table 7.9.

Table 7.9: Payoff of Bull Put Spread

Value of
Stock Price at Profit/Loss to Long Profit/Loss to Short
Combined
Maturity (RM) RM19.50 put @ RM0.20 RM20.00 put @ RM0.60
Position
15 4.3 (4.4) (0.10)
17 2.3 (2.4) (0.10)
19 0.30 (0.40) (0.10)
19.6 (0.20) 0.20 0
21 (0.20) 0.60 0.40
23 (0.20) 0.60 0.40
25 (0.20) 0.60 0.40

Lastly, we can transform Table 7.9 into a graph as shown in Figure 7.6.
TOPIC 7 TRADING STRATEGIES INVOLVING OPTIONS  93

Figure 7.6: Illustration of bull put spread

7.3.3 Summary of Bull Spreads


Lastly, let us look at the key features of a bull call or bull put spread (refer to
Table 7.10).

Table 7.10: Summary of Bull Spreads

Strategy Break-even Point Maximum Loss Maximum Profit


Bull call Lower exercise price + Difference in Difference in exercise
spread Difference in premium premium price ă Difference in
premium
Bull put Higher exercise price ă Difference in Difference in premium
spread Difference in premium exercise price ă
Difference in
premium
When to use When one is neutral to bullish or moderately bullish about the
underlying asset price.
Risk profile Limited loss, limited profit.
Break-even As described above.
point
Desired To take advantage of expected marginal upward movement while also
objective limiting downside risk.
94  TOPIC 7 TRADING STRATEGIES INVOLVING OPTIONS

7.3.4 Bear Call Spread


A bear spread is an appropriate strategy when one is mildly bearish or neutral to
bearish. Let us look at an example in Illustration 7.4.

Illustration 7.4:
Suppose you are neutral to bearish about Gombak CorporationÊs stock. You want
to make some money without exposing yourself to large potential losses if the
stock price in fact goes up. The following 60 days call options on Gombak
Corporation stock are available:

(a) Gombak Corporation, RM9.50 call @ RM0.90

(b) Gombak Corporation, RM10.50 call @ RM0.15

Appropriate bear call spread would be established by following strategy:

(a) Short RM9.50 call @ RM0.90

(b) Long RM10.50 call @ RM0.15

Then, the payoff of bear call spread is calculated as shown in Table 7.11.

Table 7.11: Payoff of Bear Call Spread

Profit/Loss to Short Profit/Loss to Long


Stock Price at Value of Combined
RM9.50 call @ RM10.50 call @
Maturity (RM) Position
RM0.90 RM0.15
7 0.90 (0.15) 0.75
8 0.90 (0.15) 0.75
10 0.40 (0.15) 0.25
10.25 0.15 (0.15) 0
11 (0.60) 0.35 (0.25)
13 (2.60) 2.35 (0.25)
15 (4.60) 4.35 (0.25)

The result that we get in Table 7.11 can be graphed as in Figure 7.7.
TOPIC 7 TRADING STRATEGIES INVOLVING OPTIONS  95

Figure 7.7: Illustration of bear call spread

7.3.5 Bear Put Spread


The strategy for a bear put spread is established by going long on the higher
exercise put and shorting the lower exercise one.

Illustration 7.5:
Using the earlier example of Gombak Corporation stock, suppose the following
puts are available:

(a) Gombak Corporation, RM9.50 put @ RM0.50

(b) Gombak Corporation, RM10.50 put @ RM0.80

Appropriate bear put spread would be established by the following strategy:

(a) Short RM9.50 put @ RM0.50

(b) Long RM10.50 put @ RM0.80

The calculation for payoff of bear put spread is shown in Table 7.12.
96  TOPIC 7 TRADING STRATEGIES INVOLVING OPTIONS

Table 7.12: Payoff of Bear Put Spread

Value of
Stock Price at Profit/Loss to Short Profit/Loss to Long
Combined
Maturity (RM) RM9.50 put @ RM0.50 RM10.50 put @ RM0.80
Position
7 (2.00) 2.70 0.70
8 (1.00) 1.70 0.70
9 0 0.70 0.70
10.2 0.50 (0.50) 0
11 0.50 (0.80) (0.30)
13 0.50 (0.80) (0.30)
15 0.50 (0.80) (0.30)

Then, the result in Table 7.12 can be transformed into a graph as in Figure 7.8.

Figure 7.8: Illustration of bear put spread

7.3.6 Summary of Bear Spreads


Lastly, let us summarise the key features of bear call or bear put spread (refer to
Table 7.13).
TOPIC 7 TRADING STRATEGIES INVOLVING OPTIONS  97

Table 7.13: Summary of Bear Spreads

Strategy Break-even Point Maximum Loss Maximum Profit


Bear call Lower exercise price + Difference in Difference in
spread Difference in premium exercise price ă premium
Difference in
premium
Bear put Higher exercise price ă Difference in Difference in
spread Difference in premium premium exercise price ă
Difference in
premium
When to use When one is neutral to bearish or moderately bearish about
underlying asset price.
Risk profile Limited loss, limited profit.
Break-even point As described above.
Desired objective To take advantage of expected marginal fall in underlying asset
price while limiting loss potential.

ACTIVITY 7.1

Suppose you had just gone long on one lot of company XYZÊs stock at a
price of RM20 each, for a total investment of RM200,000. You wish to
protect yourself from any short-term downside movement in price.
Suppose six months, at-the-money put options on company XYZÊs
stock are being quoted at RM0.30 each.

What is the appropriate option strategy to hedge this position? Show


the payoff profile of the appropriate strategy.

SELF-CHECK 7.2

1. Explain both hedge strategies such as portfolio insurance and


hedging exposure to a short stock position with the help of
diagrams.

2. Explain both spread strategies such as bull and bear spreads with
the help of diagrams.
98  TOPIC 7 TRADING STRATEGIES INVOLVING OPTIONS

7.4 COMBINATION STRATEGIES


What does a combination strategy mean?

A combination strategy refers to the strategy that involves the use of both
types of options (calls and puts) as part of the strategy.

The difference between the spread and a combination is that a spread uses only
one type of option, either a call or a put, whereas combinations use both call and
put options. Though combinations can be in several variants, the two most
common combination strategies are:

(a) The straddle; and

(b) The strangle.

Both these strategies are designed for extremes of volatility. Let us learn more
about these two combined strategies in the next subtopics.

7.4.1 Straddle Strategy


The straddle strategy comes in two variants, the long straddle and short straddle
(refer to Table 7.14).

Table 7.14: Variants of Straddle Strategy

Type of Straddle Strategy Purpose


Long straddle Involves the purchase (long) of both a call and a put
option on the same underlying asset, at the same exercise
price and of the same maturity.
Short straddle Involves shorting a call and a put of the same underlying
asset, exercise price and maturity.
TOPIC 7 TRADING STRATEGIES INVOLVING OPTIONS  99

A long straddle strategy is designed to profit from extreme volatility, while the
short straddle to profit from minimal volatility. These two types of straddle
strategy are further explained as follows:

(a) Illustration 7.6: Long Straddle


Siti Bhd, the countryÊs largest oil refiner, has just been subject to a hostile
takeover by Melati Bhd. You realise that this is a potentially volatile
situation. Suppose 60 days at-the-money calls and puts are priced as
follows:

(i) Siti Bhd, RM10.00 call @ RM0.20

(ii) Siti Bhd, RM10.00 put @ RM0.10

To benefit from the underlying stock volatility, you could establish a long
straddle position as follows:

(i) Long RM10.00 call @ RM0.20

(ii) Long RM10.00 put @ RM0.10

The payoff table for long straddle is shown in Table 7.15.

Table 7.15: Payoff of Long Straddle

Value of
Stock Price at Profit/Loss to Long Profit/Loss to Long
Combined
Maturity (RM) RM10.00 call @ RM0.20 RM10.00 put @ RM0.10
Position
7 (0.20) 2.90 2.70
8 (0.20) 1.90 1.7
9 (0.20) 0.9 0.70
10.2 0 (0.10) (0.10)
11 0.8 (0.10) 0.7
13 2.8 (0.10) 2.7
15 4.8 (0.10) 4.7

The result in Table 7.15 can be transformed into a graph as shown in Figure 7.9.
100  TOPIC 7 TRADING STRATEGIES INVOLVING OPTIONS

Figure 7.9: Illustration of long straddle

Lastly, the key features of long straddle is summarised in Table 7.16.

Table 7.16: Key Features of Long Straddle

Feature Description
Scenario Underlying asset likely to undergo extreme volatility
Risk profile Limited loss, unlimited profit
Break-even point Call exercise + Total premium; Put exercise ă Total
premium
Maximum loss Total premiums
Desired objective To take advantage of potential large price swings

(b) Illustration 7.7: Short Straddle


A short straddle position can be established as follows:

(i) Short RM12.00 call @ RM0.40

(ii) Short RM12.00 put @ RM0.30

Then, the payoff of short straddle is shown in Table 7.17.


TOPIC 7 TRADING STRATEGIES INVOLVING OPTIONS  101

Table 7.17: Payoff of Short Straddle

Value of
Stock Price at Profit/Loss to Short Profit/Loss to Short
Combined
Maturity (RM) RM12.00 call @ RM0.40 RM12.00 put @ RM0.30
Position
8 0.40 (3.70) (3.30)
9 0.40 (2.70) (2.30)
10 0.40 (1.70) (1.30)
11 0.40 (0.70) (0.30)
11.3 0.40 (0.40) 0
12 0.40 0.30 0.70
12.7 (0.30) 0.30 0
13 (0.60) 0.30 (0.30)
14 (1.60) 0.30 (1.30)
15 (3.60) 0.30 (3.30)

Figure 7.10 shows the graph of short straddle based on the results from
Table 7.17.

Figure 7.10: Illustration of short straddle


102  TOPIC 7 TRADING STRATEGIES INVOLVING OPTIONS

Lastly, the key features of short straddle are listed in Table 7.18.

Table 7.18: Key Features of Short Straddle

Feature Description
Scenario When minimal price movement is expected
Risk profile Limited profit, unlimited loss
Break-even point Call exercise + Total premium; Put exercise ă Total premium
Maximum profit Total premiums
Desired objective To profit from unchanged underlying asset price

7.4.2 Strangle Strategy


Strangles are used for largely the same objective and market or asset price
expectation as straddles. The difference between a straddle and strangle is that in
a strangle, the options are bought (sold) at different exercise prices. There are two
variants of strangle strategy as stated in Table 7.19.

Table 7.19: Two Variants of Strangle Strategy

Type of Strangle Strategy Purpose


Long strangle Appropriate when extreme volatility is expected to cause
the underlying asset to break-out of its trading range.
Short strangle Appropriate when minimal volatility would mean
continued range trading.

These two variants of strangle strategy are further explained as follows:

(a) Illustration 7.8: Long Strangle


Taman Bhd is a large oil and gas firm whose stock has traditionally been
trading in the RM9.50 to RM10.50 range. You believe that given the
companyÊs problems with due to oil price fluctuations, the stock price could
come under turbulence. Taman Bhd RM10.50 and RM9.50 of 60 days calls
and puts are quoted as follows:

(i) Taman Bhd, RM10.50 call @ RM0.30

(ii) Taman Bhd, RM9.50 put @ RM0.50


TOPIC 7 TRADING STRATEGIES INVOLVING OPTIONS  103

The long strangle position is established by following strategy:

(i) Long RM10.50 call @ RM0.30

(ii) Long RM9.50 put @ RM0.50

The payoff table for strangle strategy is shown in Table 7.20.

Table 7.20: Payoff of Strangle Strategy

Value of
Stock Price at Profit/Loss to Long Profit/Loss to Long
Combined
Maturity (RM) RM10.50 call @ RM0.30 RM9.50 put @ RM0.50
Position
7 (0.30) 2.00 1.70
8 (0.30) 1.00 0.70
8.7 (0.30) 0.30 0
9 (0.30) 0 (0.30)
10 (0.30) 0 (0.30)
11 0.20 (0.50) (0.30)
11.3 0.50 (0.50) 0
12 1.20 (0.50) 0.70
13 2.20 (0.50) 1.70
15 4.20 (0.50) 2.70

Then, the graph for the long strangle is shown in Figure 7.11.
104  TOPIC 7 TRADING STRATEGIES INVOLVING OPTIONS

Figure 7.11: Illustration of strangle strategy

Lastly, Table 7.21 shows you the key features of long strangle.

Table 7.21: Key Features of Strangle Strategy

Feature Description
Scenario Underlying asset price expected to breakout of current trading
range
Risk profile Limited loss, unlimited profit
Break-even point Call exercise + Total premium; Put exercise ă Total premium
Maximum loss Total premiums
Desired objective To take advantage of extreme volatility causing a price breakout
from range

(b) Illustration 7.9: Short Strangle


You have just read that given current inventory, expected supply and
demand, palm oil prices are expected to remain stable over at least the next
three months.
TOPIC 7 TRADING STRATEGIES INVOLVING OPTIONS  105

Given this information, you believe that the Wangsa Group (KLK), a large
oil palm plantation, will continue trading in its current RM9.50 to RM10.50
range. The 60 days, RM9.50 put and RM10.50 call are quoted as follows:

(i) Wangsa, RM10.50 call @ RM0.30

(ii) Wangsa, RM9.50 put @ RM0.50

The short strangle is established by the following strategy:

(i) Short RM10.50 call @ RM0.30

(ii) Short RM9.50 put @ RM0.50

The payoff table for short strangle is shown in Table 7.22.

Table 7.22: Payoff of Short Strangle

Value of
Stock Price at Profit/Loss to Short Profit/Loss to Short
Combined
Maturity (RM) RM10.50 call @ RM0.30 RM9.50 put @ RM0.50
Position
7 0.30 (2.00) (1.70)
8 0.30 (1.00) (0.70)
8.70 0.30 (0.30) 0
9 0.30 0 0.30
10 0.30 0 0.30
11.30 (0.50) 0.50 0
12 (1.20) 0.50 (0.70)
13 (2.20) 0.50 (1.70)
15 (4.20) 0.50 (3.70)

Figure 7.12 shows the graph for the short strangle.


106  TOPIC 7 TRADING STRATEGIES INVOLVING OPTIONS

Figure 7.12: Illustration of short strangle

Lastly, the key features of short strangle is given in Table 7.23.

Table 7.23: Key Features of Short Strangle

Feature Description
Scenario Underlying asset expected to continue trading in its current
range
Risk profile Limited profit, unlimited loss
Break-even Call exercise + Total premium; Put exercise ă Total premium
point
Maximum profit Total premiums
Desired To take advantage of underlying asset price remaining within
objective range
TOPIC 7 TRADING STRATEGIES INVOLVING OPTIONS  107

SELF-CHECK 7.3

1. What is a combination strategy?

2. Explain both straddle and strangle strategies with the help of


diagrams.

7.5 STRATEGIES BY MARKET OUTLOOK


Before we end this topic, let us look at different strategies by market outlook in
graphical forms so that you can easily understand these strategies (refer to
Table 7.24).

Table 7.24: Strategies by Market Outlook

Strategy Variant

Bullish

Neutral to
bullish

Bearish

Neutral to
bearish
108  TOPIC 7 TRADING STRATEGIES INVOLVING OPTIONS

Neutral
(minimum
volatility)

Extreme
volatility

Source: Bacha (2012)

 The four types of option strategies are uncovered or naked positions, hedge
positions, spreads and combination strategies.

 An uncovered or ‰naked‰ position is where one takes a position in an asset


without establishing an offsetting position.

 A hedge strategy combines an option with the underlying asset in such a way
that the overall position either reduces or eliminates risk. A fully hedged
position is riskless. The combination is such that price movements offset each
other.

 A spread strategy can be thought of as a speculative position with a safety


net. A spread position typically involves the establishment of offsetting
positions in the same asset but across different markets, at different
maturities or different exercise prices.

 A combination strategy involves the use of both types of options, calls and
puts as part of the strategy. The two most common combination strategies are
straddle and strangle.

 Different strategies by market outlook are bullish, neutral to bullish, bearish,


neutral to bearish, neutral (minimum volatility) and extreme volatility.
TOPIC 7 TRADING STRATEGIES INVOLVING OPTIONS  109

Bear call spread Long straddle

Bear put spread Long strangle

Bear spread Portfolio insurance

Bull call spread Short straddle

Bull put spread Short strangle

Bull spread Spread strategies

Combination strategies Stock position

Conversion strategy Straddle

Hedge strategies Strangle

Hedging exposure to a short Uncovered or naked positions

Bacha, O. I. (2012). Financial derivatives: Markets and applications in Malaysia


(3rd ed.). Shah Alam, Malaysia: McGraw-Hill.

Hull, J. C. (2012). Options, futures, and other derivatives (8th ed.). Boston, MA:
Prentice Hall.
Topic  Option Pricing
8 Models

LEARNING OUTCOMES
By the end of the topic, you should be able to:
1. Calculate the option prices by using binomial option pricing model
(BOPM) for call options;
2. Find the probabilities and volatility changes;
3. Calculate the option prices by using binomial option pricing model
(BOPM) for put options;
4. Find the option prices by applying the Black-Scholes option pricing
model (BSOPM);
5. Identify the determinants of option prices; and
6. Summarise the implied volatilities.

 INTRODUCTION
In this topic, we are going to look at how to price option instruments as it is
another important aspect of options. There are two common methods to price
options such as binomial option pricing model (BOPM) and the Black-Scholes
option pricing model (BSOPM). We will look into each of them one by one as
well as the probabilities and volatility changes, determinants of option prices and
implied volatilities. I hope you are ready to discover more on these matters. Let
us continue the lesson.
TOPIC 8 OPTION PRICING MODELS  111

ACTIVITY 8.1

What are two common methods to price options?

8.1 BINOMIAL OPTION PRICING MODEL


(BOPM) – CALL OPTION
The binomial option pricing model is according to the logic that the current value
of the option is equal to the present value of the possible payoffs to the option at
maturity (Bacha, 2012; Hull, 2012). The BOPM is a discrete time model, in that
underlying asset price changes at a given fixed time interval. Let us look at an
illustration to understand more on this matter.

Illustration 8.1:
Suppose we want to find the value of a European style call option on an
underlying stock which is currently selling at RM20 with the following
assumptions (refer to Table 8.1).

Table 8.1: Assumptions of a European Style Call Option

Option types The call option on the stock has a RM20 exercise price and
one year maturity.
Price changes Only change in price once during the one year.
Price movement The percentage change in the stockÊs price is 20%, that is, it
can either go up or down by a fixed 20%.
Probability The probability of an up or down movement is an equal 50%.
Risk-free interest rate 10% per annum.

The possible stock and call values at maturity can be simplified into Figure 8.1.

Figure 8.1: Single period binomial option pricing model for call options
112  TOPIC 8 OPTION PRICING MODELS

At maturity in one year, the stockÊs price could either be 20% higher or lower.
The probabilities of each outcome occurring is 50% or 0.5. Given this, the call
denoted Ct now, would have a payoff of RM4 (Stock price ă Exercise price) or
RM0. The call is only valuable if it ends in-the-money. Since the probability of the
stock going up is 50%, the RM4.00 payoff from the call has a 50% probability.

Using single period binomial option pricing model, the value of the call is
RM1.82 (refer to Table 8.2).

Table 8.2: Calculation for Illustration 8.1

Formula Calculation
Pu  C u  Pd  C d 0.5  4  0.5  0
Ct = C1yr =
(1  r )t (1  0.1)1
= RM1.82

8.1.1 Two-step Binomial Trees


Holding all other assumptions are same, suppose we now allow the stockÊs price
to change twice within a year, that is, once every six months; the stock price
movement and corresponding payoff would be as shown in Figure 8.2.

Figure 8.2: Two-step binomial tree for call option


TOPIC 8 OPTION PRICING MODELS  113

Then,

C2 = 0.25  8.8  0.25  0  0.25  0  0.25  0


 RM1.9955
(1  0.05) 2

The value of call using BOPM would be RM1.9955.

8.1.2 Three-step Binominal Trees


Suppose we now relax the periodic price change assumption and allow the stock
price to change at three times per year (see Figure 8.3).

Figure 8.3: Three-step binomial tree for call option

Then,

0.125  14.56  0.125  3  3.04  0.125  3  0  0.125  0


C3 
(1  0.1/ 3) 3
1.82  1.14
  RM 2.68
(1  0.1/ 3) 3

How do we get 0.125? We get this number by following the tree: 0.50.50.5 =
0.125. The second component needs to multiplied by three; the trees will be in
three conditions such as „UP UP DOWN‰, „UP DOWN UP‰ and „DOWN UP
UP‰. Therefore, the value of the call would be RM2.68 by using BOPM.
114  TOPIC 8 OPTION PRICING MODELS

ACTIVITY 8.2

Find the value of a European style call option on an underlying stock


which is currently selling at RM10 with the following assumptions:

(a) The call option on the stock has a RM10 exercise price and one
year maturity;

(b) Change in price three times during the one year;

(c) The percentage change in the stockÊs price is 10%, that is, it can
either go up or down by a fixed 10%;

(d) The probability of an up is 60% and down movement is an equal


40%; and
(e) Interest rate is 8% per annum.

8.2 PROBABILITIES AND VOLATILITY


In the previous subtopic, we have looked at only the price changes during the
given time period. Therefore, this subtopic will discuss the probability and
volatility changes during the given time period.

8.2.1 Probability Changes


In scenarios till now, probability has been assumed homogenous across all
possibilities. However, this may not be the case always, as a bullish opinion,
would lead to higher weightage assigned to positive outcomes. For example:

0.216  14.56  0.144  3  3.04  0  0 3.145  1.313


C3 = 
(1  0.1/ 3) 3 (1  0.1/ 3) 3
4.458
=  RM 4.04
(1  0.1/ 3) 3

The three-step binomial trees with probability changes are shown in Figure 8.4.
TOPIC 8 OPTION PRICING MODELS  115

Figure 8.4: Three-step binomial trees with probability changes

In a three period scenario where probabilities for the alternative paths are not
equal anymore, the value of the call is different than the equal probability
weighted scenario. Therefore, the value of the call would be RM4.04 in this case.

8.2.2 Volatility Changes


In earlier scenarios we assumed that the underlying stock price would change by
20%. Suppose we now increase the volatility to 30%. How would the call value
change? Let us look at the diagram of a three-step binomial tree with volatility
changes in Figure 8.5.

Figure 8.5: Three-step binomial tree with volatility changes


116  TOPIC 8 OPTION PRICING MODELS

The calculation would be

0.125  23.94  0.125  3  3.66  0  0 2.9925  1.3725


C3 = 
(1  0.1/ 3) 3 (1  0.1/ 3) 3
4.365
=  RM 3.956
(1  0.1/ 3) 3

With 10% increase in volatility from 20% to 30%, the call option value goes from
RM2.68 to RM3.956. This is how volatility impacts the value of call options.

8.3 BINOMIAL OPTION PRICING MODEL


(BOPM) – PUT OPTION
Keep in mind that the way pricing put options are calculated is exactly the same
as that of the valuation of calls. In our earlier illustration, we assume a put option
now instead of a call option.

0  0  0.125  3  4.64  0.125  9.76 1.74  1.22


C3 = 
(1  0.1/ 3) 3 (1  0.1/ 3) 3
= RM2.68

Then, the three-step binomial tree for the put option is shown in Figure 8.6.

Figure 8.6: Three-step binomial tree for put option

Volatility and probability would impact the put option price in a similar fashion
as the impact on call options. However, we are not going to elaborate on these
TOPIC 8 OPTION PRICING MODELS  117

scenarios. But, you are encouraged to do so on your own in order to better


understand the impact of probability and volatility on put options.

SELF-CHECK 8.1

Explain the binominal option pricing model.

8.4 THE BLACK-SCHOLES OPTION PRICING


MODEL (BSOPM)
Now we come to the Black-Scholes option pricing model (BSOPM). The BSOPM
has applications in various areas beyond option pricing alone. Its authors, Fisher
Black and Myron Scholes, were awarded the Nobel Prize for Economics in 1995.

What is the advantage of the BSOPM over other models? The biggest advantage
of the BSOPM over other models is that the BSOPM provides a closed-form
solution to option pricing. This model is in a continuous time form as it is
opposite to the binominal pricing model where the model is based on discrete
time form; meaning that the time interval between underlying asset price change
is as small as to approach zero.

8.4.1 Underlying Assumptions of the Black-Scholes


Option Pricing Model (BSOPM)
What are the underlying assumptions of the BSOPM? The underlying
assumptions of the BSOPM are as follows:

(a) Efficient markets with frictionless trading.

(b) No transaction costs (the model ignores bid-ask spread, commissions and
so on).

(c) Option has European style exercise.

(d) The underlying stock will pay no dividends during the maturity of the
option.

(e) Underlying logarithmic stock returns are normally distributed.

(f) The risk-free interest rate remains unchanged over option maturity.

(g) Underlying stock volatility is constant over option maturity.


118  TOPIC 8 OPTION PRICING MODELS

Of all these assumptions, the last two of unchanged interest rates and constant
volatility of the underlying asset until option maturity are considered the most
restrictive.

8.4.2 The Black-Scholes Option Pricing Model


(BSOPM) for a Call Option
The Black-Scholes option pricing model formula for a call option is as follows:

C  S.N (d 1 )  Ke  rt  N (d 2 )

How do we get d1 and d2? We can use the following formula (refer to Table 8.3).

Table 8.3: Formula for d1 and d2

d1 d2

S  2  d 2  d1   T
ln( )   r  ( )  T
K  2 
d1 
 T

Where:
S = Spot price of underlying asset
K = Exercise price of call option
T = Time to expiration
r = Risk free interest rate
e-rt = Exponential function of rf interest rate and time
N(.) = Cumulative standard normal distribution (SND) function
 = Volatility of underlying asset as measured by standard
deviation
S  = Natural logarithm of S/K
ln 
K 

From the previous description, we can see that there are three steps to calculate
the price of call options (refer to Table 8.4).
TOPIC 8 OPTION PRICING MODELS  119

Table 8.4: Steps in Calculating Price of Call Options

Step Action
First step Calculate d1 and d2.
Second step Using the cumulative normal distribution table, find the values of
N(d1) and N(d2).
Third step Plug the values into the model and solve.

Let us see an example to demonstrate this matter.

Example 8.1:
Suppose:
Stock price, S = RM21
Exercise price, K = RM20
Interest rate, r = 0.08
Maturity, T = 180 days = 0.5
Standard deviation, Ĕ  = 0.5

What is the correct price of the call?

Solution:

Step 1:
d1 d2

21  0.5 2  d 2  0.43  0.35


ln( )  0.08  ( )  0.5
20  2 
d1  = 0.08
0.5 0.5

0.0488  0.205  0.5 0.1513


 
0.3535 0.3535

= 0.43

Step 2:
Look for N(d1) and N(d2) from the cumulative standard normal distribution
table:

N(d1) = N(0.43) = 0.6664


N(d2) = N(0.08) = 0.5319
120  TOPIC 8 OPTION PRICING MODELS

Step 3:
C = RM21  0.6664 ă RM20  e-0.080.5  0.5319
= RM13.99 ă RM19.22  0.5319 = RM13.99 ă RM10.22 = RM3.77

Decomposing the previous call value of RM3.77 into intrinsic and time values,
the intrinsic value here is RM1 while the remainder RM2.77 would constitute
time value.

8.4.3 The Black-Scholes Option Pricing Model


(BSOPM) for a Put Option
Though the BSOPM was developed to price European call options, the model can
just as easily be used in valuing European style put option. Thus, we can employ
the following formula:

P  Ke  rt  N (-d 2 )  S .N (-d 1 )

The steps involved in valuation are similar to earlier steps for call options.

Illustration 8.2:
Continuing with Example 8.1, we computed the price of the call option to be
RM3.77.

The first two steps:

(a) Calculating d1, d2; and

(b) Finding N(d1) and N(d2) are the same.

However, a small adjustment has to be made before we plug-in and solve for
option value in Step 3. The small adjustment is to convert N(d1) and N(d2) to N(-
d1) and N(-d2).

As N(d1) was calculated earlier for the call to be 0.6664. Therefore:

N(-d1) = 1 ă N(d1) = 1 ă 0.6664 = 0.3336


TOPIC 8 OPTION PRICING MODELS  121

In a same way, since N(d2) was 0.5319:

N(-d2) = 1 ă N(d2) = 1 ă 0.5319 = 0.4681

P = 20  e-0.080.5  0.4681 ă RM21  0.3336

= RM19.22  0.4681 ă RM7.01= RM8.00 ă RM7.01 = RM0.99

The put option price is RM0.99.

SELF-CHECK 8.2

Explain the Black-Scholes options pricing model with one example.

8.5 DETERMINANTS OF OPTION PRICES


In the Black-Scholes options pricing model (BSOPM), there are five related input
variables as stated in Figure 8.7.

Figure 8.7: Five determinants of option prices

These five variables are further explained as follows:

(a) Stock/Underlying Asset Price


The change in stock or underlying asset price is positively correlated to call
values and negatively to puts. This relationship between the underlying
asset price and option values are known as deltas.

C Change in value of option


Delta  
S Change in underlying asset value
122  TOPIC 8 OPTION PRICING MODELS

(b) Exercise Price


The exercise price has a negative correlation with call options and a positive
one with put options. Raising the exercise price benefits put options but
works against calls.

(c) Volatility
Underlying asset price volatility has a positive correlation with both option
prices. The relationship between option value and underlying asset
volatility is known as vega.

C Change in value of option


Vega  
 Change in underlying asset price volatility

(d) Interest Rates


The impact of interest rates on option values can be seen directly from the
intrinsic value equations. An increase in interest rates would increase the
call intrinsic value since the present value of the exercise price would be
lower.

However, in the case of put options, the effect is opposite; higher interest
rates reduce the present value of exercise price. This relationship between
interest rates and option values is termed as rho.

C Change in value of option


Rho  
r Change in interest rate

(e) Time to Maturity


The relationship between time to maturity and option values is termed as
theta. Time to maturity is positively correlated with calls but has an
ambiguous relationship to put values. This is due to the opposite impact of
time to maturity on the put intrinsic and time values.

C Change in value of option


Theta  
t Change in time to maturity

SELF-CHECK 8.3

What are the determinants of option prices?


TOPIC 8 OPTION PRICING MODELS  123

8.6 IMPLIED VOLATILITIES


Lastly, let us look at implied volatility. In a simple definition, implied volatility is
the volatility implied in an option price. As explained earlier, there are five
parameters that go into the BSOPM to determine the option value.

However, it is possible to work out the sixth variable given any five variables.
Thus, given the four other input variables (s, k, r and T) and the call value, we
can derive the volatility estimate that justifies the given call value. This would be
the implied volatility.

There are two common uses of implied volatility in option trading which are:

(a) Traders can use implied volatility estimates to determine the


‰expensiveness‰ of an option relative to other options.

(b) To determine option mispricing. A rule of thumb use by traders is to


compare implied volatility with actual or historical volatility.

 The binomial option pricing model (BOPM) is a discrete time model in that
the underlying asset price changes at a given fixed time interval. It is
according to the logic that the current value of the option is equal to the
present value of the possible payoffs to the option at maturity.

 Probability has been assumed homogenous across all possibilities. However,


this may not be the case always, as a bullish opinion would lead to higher
weightage assigned to positive outcomes.

 Volatility impacts the value of a call option.

 Volatility and probability would impact the put option price in a similar
fashion as that on call options.

 The Black-Scholes option pricing model (BSOPM) is based on a discrete time


form; the time interval between underlying asset price change is as small as
to approach zero. It provides a closed-form solution to option pricing.

 There are five determinants of option prices namely (i) stock/underlying


asset price, (ii) exercise price, (iii) volatility, (iv) interest rates and (v) time to
maturity.
124  TOPIC 8 OPTION PRICING MODELS

 Implied volatility is the volatility implied in an option price. Two common


uses of implied volatility in option trading are to determine the
‰expensiveness‰ of an option relative to other options and to determine
option mispricing.

Binominal option pricing model Probability changes


(BOPM)
Three-step binominal trees
Black-Scholes option pricing model
(BSOPM) Two-step binominal trees

Determinants of option prices Volatility changes

Implied volatilities

Bacha, O. I. (2012). Financial derivatives: Markets and applications in Malaysia


(3rd ed.). Shah Alam, Malaysia: McGraw-Hill.

Hull, J. C. (2012). Options, futures, and other derivatives (8th ed.). Boston, MA:
Prentice Hall.
T op i c  Options on
9 Stock Indices,
Currencies
and Futures
LEARNING OUTCOMES
By the end of the topic, you should be able to:
1. Apply options on single stocks;
2. Examine the application of options on stock indices;
3. Apply stock index options on portfolio insurance; and
4. Use currency and futures options.

 INTRODUCTION
We have demonstrated the application of options solely on single stocks. Now it
is time for you to understand how options are applied to stock indices, portfolio
insurance, currencies and futures. Are you ready? Let us start the lesson.
126  TOPIC 9 OPTIONS ON STOCK INDICES, CURRENCIES AND FUTURES

9.1 STOCK FOR PAYING A KNOWN DIVIDEND


YIELD
We will get the same probability distribution for the stock price at time T in the
following scenario. This is similar to the concept with bonds, where a par bond is
with coupon and a discounted bond does not necessarily have a coupon:

(a) The stock starts at price S0 and offers a dividend yield = q; and

(b) The stock starts at price S0 eqT and there is no income.

Therefore, we are able to value European options by decreasing the stock price to
S0 e-qT and then treating it as not providing dividend.

If we recall in the earlier topic, one of the major assumptions of the Black-Scholes
option pricing model (BSOPM) is that the underlying stock will pay no dividends
during the maturity of the option (Bacha, 2012). Can you still recall? Now let us
try to modify the formula for those stocks with dividends.

c  S 0e -qT N (d 1 )  Ke - rT N (d 2 )
p  Ke - rT N (-d 2 )  S 0e -qT N (-d 1 )
ln(S 0 / K )  ( r  q   2 / 2)T
where d 1 
 T
ln(S 0 / K )  ( r  q   2 / 2)T
d2 
 T

Can you notice that it is similar to the Black-Scholes option pricing model
(BSOPM)? However, the main difference lies in the dividend which is denoted as
q.

9.2 STOCK INDEX OPTIONS


So far, we just looked at the options on single stocks. Now let us examine the
application of options on stock indices. Bear in mind that the logic behind both
stock and stock index options are the same. The only difference is that for stock
index options, the underlying asset will be stock indices or basket of stocks. The
most common stock index options are the Dow Jones Industrial (European) DJX,
the S&P 100 (American) OEX and the S&P 500 (European) SPX. Similar to other
options, stock index options are also settled in cash.
TOPIC 9 OPTIONS ON STOCK INDICES, CURRENCIES AND FUTURES  127

In Malaysia, the first traded option was introduced in year 2000 by Kuala
Lumpur Options and Financial Futures Exchange (KLOFFE) (later merged and
named Bursa Malaysia Derivatives Bhd). This first option contracts were FBM
KLCI Options with FTSE Bursa Malaysia, Kuala Lumpur Composite Index as the
underlying asset. The FBM KLCI Options have both call and put options of
varying exercise prices. Let us look at an example that demonstrates stock index
options in Illustration 9.1.

Illustration 9.1:
Suppose a trader is bullish about the FBM KLCI and wishes to trade options to
benefit from a potential upside rally. The FBM KLCI is now 1,700 points and 60
days at-the-money index calls are being quoted as follows:

1,700 call @ 10 points

This means that the premium in ringgit will be 10 points and the index multiplier
is RM50. To purchase the call, the investor pays a premium of 10 points  RM50
= RM500.

Let us look at two different scenarios in Table 9.1.

Table 9.1: Two Different Scenarios for Stock Index Options

Scenario 1 Scenario 2
FBM KLCI goes up by 30 points: FBM KLCI goes down by 30 points:
 At maturity, the FBM KLCI will be  At maturity, the FBM KLCI will be
1,730 points. 835 points.
 The call option is clearly in-the-money  The call option is out-of-the-money
and profitable for the holder to and will not be exercised.
exercise.

Profit on exercise for calls


 (Settlement value ă Exercise price) ï
Index multiplier
 (1,730 points ă 1,700 points)  RM50
 (30 points)  RM50 = RM1,500
128  TOPIC 9 OPTIONS ON STOCK INDICES, CURRENCIES AND FUTURES

The summary for these two scenarios are as follows (refer to Table 9.2 and
Table 9.3):

Table 9.2: Summary for Scenario 1

Long Call (Buyer of Call Option) Short Call (Seller of Call Option)
Pay premium (RM500) Receive premium RM500
Profit from exercise RM1,500 Loss from exercise (RM1,500)
Net profit RM1,000 Net loss (RM1,000)

Table 9.3: Summary for Scenario 2

Long Call (Buyer of Call Option) Short Call (Seller of Call Option)
Pay premium (RM500) Receive premium RM500
Profit from exercise 0 Loss from exercise 0
Net profit (RM500) Net loss RM500

The maximum loss possible to the long position in options is the amount of
premium paid. In other words, the maximum loss for the buyer of the call option
is the premium paid.

SELF-CHECK 9.1

Explain the application of options on the stock indices with one


example.

9.3 USING STOCK INDEX OPTIONS FOR


PORTFOLIO INSURANCE
The fund manager holds the portfolio insurance. In holding the portfolio, the
fund manager faces risks if the value of portfolio goes down. Therefore, it is
requited that the fund manager subscribe to index puts options.

We can calculate how many contracts to buy to fully protect the portfolio using
the following formula:

No. index puts required = Value of holding/(Index level  Contract multiplier)


TOPIC 9 OPTIONS ON STOCK INDICES, CURRENCIES AND FUTURES  129

Let us look at Example 9.1 which demonstrates this matter.

Example 9.1:
A Malaysian fund manager is holding a combined value of RM20 million.
Worried by the news of oil price downturn, the fund manager decides to insure
his portfolio holding by buying FBM KLCI index put which expires in three
months (90 days) time in June. The current level of FBM KLCI is 1,700 and the
JUNE 1675 FBM KLCI put contract costs RM20. The FBM KLCI option has a
contract multiplier of RM50. Find the number of contracts needed.

Solution:
The number of contracts needed is:

No. index puts required = RM20,000,000/(1,700  50) = 235 contracts


Total cost of the options is: 235  RM20  RM50 = RM235,000

You can refer to Table 9.4 for the illustration of portfolio insurance by using stock
index options.

Table 9.4: Illustration of Portfolio Insurance by Using Stock Index Options

FBM KLCI Portfolio Value (RM) Put Option (RM) Insured Portfolio
Level (a) (b) (RM) (a) + (b)
1,400 16,475,000* 2,996,250* 19,471,250
1,500 17,650,000 1,821,250 19,471,250
1,600 18,825,000 646,250 19,471,250
1,700 20,000,000 (235,000) 19,765,000
1,800 21,175,000 (235,000) 20,940,000
1,900 22,350,000 (235,000) 22,115,000
2,000 23,525,000 (235,000) 23,290,000

(a) * 20,000,000 ă (1,700 ă 1,400)  235  50 = 16,475,000

(b) * (1,675 ă 1,400)  100  50 ă 2,350,000 = 2,996,250

SELF-CHECK 9.2

Explain the portfolio insurance process by using stock index options.


130  TOPIC 9 OPTIONS ON STOCK INDICES, CURRENCIES AND FUTURES

9.4 CURRENCY OPTIONS


What do currency options mean?

A currency option is a contract that permits the holder the right, but not the
obligation to purchase or sell currency at a specified exchange rate during a
specified period of time, just like the features of other option contracts.

When do we apply currency options? Corporations or individuals can apply


currency options to hedge against adverse movements in exchange rates. Let us
look at two illustrations that demonstrate currency options.

Illustration 9.2: Importer (Currency Payment)


Let us look at Table 9.5 which summarises the currency options from importer
perspective.

Table 9.5: Currency Options from Importer Perspective

Suppose a Malaysian importer has to pay USD3 million sometime during the next six
months. To hedge this, the importer buys a call option on the USD and the option
premium is RM0.015/USD, for options with K = RM4.00/USD
What option should the importer Since the importer has to make a USD
purchase? payment, he should buy options that give
him the right to buy USD: CALL options
Premium paid 3m  0.015 = RM45,000
What is the maximum that the importer The max that he will have to pay for each
has set on the price of the USD? USD is RM0.015/USD + RM4.00/USD =
RM4.015/USD
What is the actual amount that the Since ST < K, the options are worthless
importer will pay if the spot rate at the and the importer can do better by buying
end of six months is RM3.80/USD? at the market rate of RM3.8/USD. Thus,
his total cost, ignoring time value of the
payments, is RM0.015/USD + RM
3.80/USD = RM3.815/USD
What is the actual amount that the Now, ST > K. Therefore, it is worth
importer will pay if the spot rate at the exercising the options. The importer will
end of six months is RM4.2/USD? pay his maximum price, RM4.015/USD

Source: Hull (2012)


TOPIC 9 OPTIONS ON STOCK INDICES, CURRENCIES AND FUTURES  131

Illustration 9.3: Exporter (Receive Currency)


As for Table 9.6, it shows you the summary of currency options from the
exporterÊs perspective.

Table 9.6: Currency Options from ExporterÊs Perspective

Suppose a Chinese company, Tianshan, has to sell USD10 million sometime during the
next three months, and would like to lock-in a minimum RMB value for this. The price
of a put option with a strike price of K = RMB6.50/USD is RMB0.05/USD.
What option should the exporter buy? Since Tianshan is going to sell USD, it should
buy a put option on the USD. This is, of
course, the same as wanting to buy RMB.
Therefore, a call option on the RMB: PUT
options
Premium paid USD30 m  RMB0.05/USD = RMB1.5 m
What is the floor that the Tianshan has The min that they will have to receive for each
set on the price of the USD? USD is:
= K ă Premium
= RMB6.5 ă RMB0.05 = RMB6.45
What is the actual amount that the Since ST > K, the options are worthless and
company receives if the spot rate at the Tianshan can do better by selling at the
end of three months is RMB6.7/USD? market rate of RMB6.7/USD, rather than the
exercise price of RMB6.5/USD. Thus, their
total receipts will be
= RMB6.7/USD ă RMB0.05/USD
= RMB6.65/USD
What is the actual amount that the Now, ST < K. Therefore, it is worth exercising
company receives if the spot rate at the the options. Tianshan will receive their floor
end of three months is RMB6.2/USD? price, RMB6.5 ă RMB0.05 = RMB6.45

Source: Hull (2012)

9.4.1 Valuing European Currency Options


We denote the foreign interest rate by rf. When a US company buys one unit of
the foreign currency, it has an investment of S0 dollars. The return from investing
at the foreign rate of rf S0 dollars. This shows that the foreign currency provides
a „dividend yield‰ at rate rf . A foreign currency is an asset that provides a
„dividend yield‰ equal to rf. We can use the formula for an option on a stock
paying a dividend yield:
Set S0 = Current exchange rate
Set q = rf
132  TOPIC 9 OPTIONS ON STOCK INDICES, CURRENCIES AND FUTURES

Therefore, the modified formula will be:

c  S 0e  rf T N (d1 )  Ke  rT N (d 2 )
p  Ke  rT N ( d 2 )  S 0e  rf T N ( d1 )
ln(S 0 / K )  ( r  rf   2 / 2)T
where d1 
 T
ln(S 0 / K )  ( r  rf   2 / 2)T
d2 
 T

SELF-CHECK 9.3

1. Explain the currency options from the perspectives of the


importer and exporter.

2. How does one value European currency options?

9.5 FUTURES OPTIONS


We have already explained options on stock indices and currency. Now as the
last subtopic, we are going to explain the futures options. Similarly, there are two
different options available for futures as well (refer to Table 9.7).

Table 9.7: Two Different Mechanics of Call and Put Futures Options

Mechanics of Call Futures Options Mechanics of Put Futures Option


When a call futures option is exercised When a put futures option is exercised
the holder acquires: the holder acquires:
(a) A long position in the futures; and (a) A short position in the futures; and
(b) A cash amount equal to the excess of (b) A cash amount equal to the excess of
the futures price over the strike price. the strike price over the futures
price.
Payoff from call = F0 ă K Payoff from put = K ă F0
Where F0 is futures price at time of exercise
TOPIC 9 OPTIONS ON STOCK INDICES, CURRENCIES AND FUTURES  133

9.5.1 Valuing European Futures Options


We can use the formula for an option on a stock paying a dividend yield by:

Setting S0 = Current futures price (F0)


Setting q = Domestic risk-free rate (r )
Setting q = r ensures that the expected growth of F in a risk-neutral
world is zero.

A futures contract requires no initial investment. In a risk-neutral world, the


expected return should be zero. The expected growth rate of the futures price is,
therefore, zero. The futures price can therefore be treated like a stock paying a
dividend yield of r, and then we use the same Black-Scholes formula to derive
this:

c  e  rT F 0 N (d 1 )  K N (d 2 )
p  e rT K N (d 2 )  F 0 N (d 1 )
ln(F 0 / K )   2T / 2
where d 1 
 T
ln(F 0 / K )   2T / 2
d2   d1   T
 T

As a summary, we can treat stock indices, currencies and futures like a stock
paying a dividend yield of q (see Figure 9.1).

Figure 9.1: Summary for stock indices, currencies


and futures of a stock paying a dividend yield of q
134  TOPIC 9 OPTIONS ON STOCK INDICES, CURRENCIES AND FUTURES

 The options on single stocks can be calculated by using this formula:


c  S 0e qT N (d 1 )  Ke  rT N (d 2 )
p  Ke rT N ( d 2 )  S 0e qT N (d 1 )
ln(S 0 / K )  (r  q   2 / 2)T
where d 1 
 T
ln(S 0 / K )  (r  q   2 / 2)T
d2 
 T

 The stock index options underlying asset will be stock indices or basket of
stocks. The most common stock index options are the Dow Jones Industrial
(European) DJX, the S&P 100 (American) OEX and the S&P 500 (European)
SPX.

 Fund managers need to subscribe to index puts options in order to avoid or


minimise the risks if the value of portfolio goes down.

 The formula to calculate how many contracts to buy to fully protect the
portfolio is:
No. index puts required = Value of holding/(Index level  Contract
multiplier)

 A currency option contracts is a contract that permits the holder the right, but
not the obligation to purchase or sell currency at a specified exchange rate
during a specified period of time. Corporations or individuals can apply
currency options to hedge against adverse movements in exchange rates.

 There are two different options available for futures options, namely,
mechanics of call futures options and mechanics of put futures option.

Currency options Stock indices options

Futures options Valuing European futures options

Portfolio insurance Valuing European currency options


TOPIC 9 OPTIONS ON STOCK INDICES, CURRENCIES AND FUTURES  135

Bacha, O. I. (2012). Financial derivatives: Markets and applications in Malaysia


(3rd ed.). Shah Alam, Malaysia: McGraw-Hill.

Hull, J. C. (2012). Options, futures, and other derivatives (8th ed.). Boston, MA:
Prentice Hall.
Topic  Derivative
10 Instruments
and Islamic
Finance
LEARNING OUTCOMES
By the end of the topic, you should be able to:
1. Identify the prerequisites of Islamic financial instruments;
2. Identify several Islamic financial instruments which features are
similar to derivative instruments;
3. Explain how to price a sukuk ijara with warrant or embedded
options; and
4. Summarise the current derivative instruments from the Islamic
perspective.

 INTRODUCTION
Islamic financial institutions (IFIs), like conventional financial systems, face some
risks in mitigating sharia-compliant derivatives as Islamic finance emerges in the
financial system. Malaysia (as one of pioneers of Islamic finance industry in the
world), finds that it is very useful to understand the financial derivatives in
Islamic finance.

Therefore, the objective of this last topic is to provide the essential knowledge on
permissibility and applicability of financial derivatives in Islamic finance. Are
you ready to discover more on Islamic financial instruments? Let us continue the
lesson.
TOPIC 10 DERIVATIVE INSTRUMENTS AND ISLAMIC FINANCE  137

10.1 THE PREREQUISITES OF ISLAMIC


FINANCIAL INSTRUMENTS
Generally, all Islamic financial instruments and transactions must meet a number
of prerequisites in order to be considered halal (acceptable) or permissible
(Bacha, 2012). All financial instruments and transactions must be free of at least
the following five items at basic level (refer to Table 10.1).

Table 10.1: The Prerequisites of Islamic Financial Instruments

Criteria to be
Explanation
Eliminated
Riba (usury) Usury is usually referred to as the receiving and paying of interest.
There are several forms of riba and all forms of riba are not
permissible.
Rishwah Meaning corruption.
Maysir In the case of the maysir, if we see it from a financial instrument
(gambling) viewpoint, it would be one where the outcome is exclusively
dependent on chance only ă as in gambling.
Gharar It translates into unnecessary risk, deception or intentionally induced
(unnecessary uncertainty.
risk) As far as the financial transactions are concerned, gharar can be
considered when one or both parties are uncertain about possible
outcomes of underlying contract.
Jahl It means ignorance. From a financial transaction perspective, it would
(ignorance) be unacceptable or not permissible if one party to the transaction gains
due to the other partyÊs ignorance.

Additionally, there are other several conditions to be considered in Islamic


finance such as:

(a) The sharia has some basic requirements with regards to the sale of an asset
(for example, real assets versus financial assets). Since a derivative
instrument belongs to financial assets where its value is dependent on the
underlying asset (real asset in most cases), the sharia requirements for the
validity of a sale would also be relevant;

(b) The underlying asset of the contract must be halal and permissible from the
Islamic perspective;

(c) The underlying asset or commodity must currently exist physically and in a
sellable form; and

(d) The seller should have legal ownership of the asset in its final form.
138  TOPIC 10 DERIVATIVE INSTRUMENTS AND ISLAMIC FINANCE

SELF-CHECK 10.1

Describe the prerequisites of Islamic financing instruments.

10.2 ISLAMIC FINANCIAL INSTRUMENTS


SIMILAR TO DERIVATIVE INSTRUMENTS
There are several Islamic financial instruments which features are similar to
derivative instruments (Bacha, 2012; Bacha & Mirakhor, 2015). These Islamic
financial instruments are shown in Figure 10.1.

Figure 10.1: Four Islamic financial instruments that


have similar features with derivative instruments

These four instruments are further explained in the next sections.

10.2.1 Bay’ al-Salam


Firstly, what does bayÊ al-salam or forward sale mean?

BayÊ al-salam or forward sale is an Islamic transaction whereby the seller


agrees to undertake the delivery of an underlying asset at the stipulated
future date in exchange for a consideration paid either in cash or in kind by
the purchaser immediately at a spot price.

The quantity, quality, conditions and mode of delivery of the underlying asset
must be clearly specified upfront in order to be delivered to the purchaser at the
predetermined future date. BayÊ al-salam holds an exception to the main
principle of Islamic transaction that the seller must possess the underlying asset
before it can be sold.
TOPIC 10 DERIVATIVE INSTRUMENTS AND ISLAMIC FINANCE  139

However, unlike the conventional forward contract which payment can be


deferred, the buyer in a contract of bayÊ al-salam is required to pay the entire
amount in full at the time the contract is initiated. In fact, the contract also
stipulates that the payment made by the buyer must be in cash.

However, the bayÊ al-salam contract is subject to several generally agreed


conditions as follows:

(a) Full advance payment must be made by the buyer at the time of
contracting.

(b) The buyer does not have ownership of the asset until the delivery takes
place.

(c) The underlying asset must be standardisable, easily quantifiable and of


determinate quality.

(d) The underlying asset must not constitute ribawi items such as gold, silver,
wheat and so on, where its delivery must be made simultaneously or
otherwise will be equal to riba.

(e) BayÊ al-salam contract cannot be applied for a remotely identified


underlying asset such as commodities harvested from a particular farm or
field and so on.

(f) BayÊ al-salam contracts can only be applied for fungible goods.
(g) Quantity, quality, maturity date and place of delivery must be clearly
enumerated in the bayÊ al-salam agreement.

(h) The underlying asset or commodity must be available and traded in the
markets throughout the period of contract.

Based on the perspective of capital market, the current exchange traded futures
seem to conform to the listed conditions of bayÊ al-salam except on the
requirement of full advance payment that has to be made by the buyer. Given
this unique feature of bayÊ al-salam, it is found that the bayÊ al-salam contract
closely resembles a forward contract than a futures contract.

In fact, some limitations of forward contracts such as ‰multiple-coincidence‰ that


leads to price squeeze and counterparty risks can also potentially exist in a bayÊ
al-salam contract. However, counterparty risks associated with bayÊ al-salam
contracts would only have to be borne by the buyer since a full advance payment
has been made upfront.
140  TOPIC 10 DERIVATIVE INSTRUMENTS AND ISLAMIC FINANCE

Therefore, in order to protect the interests of the buyer over any potential default
by the seller or failure of delivery, sharia allows for the buyer to ask for security
from the seller in a form of guarantee or mortgage.

In addition, a bayÊ al-salam contract can be utilised as an underlying sharia-


compliant contract in the provision of working capital financing by Islamic
financial institutions (IFIs). Yet, since the IFIs would not want to take possession
of the underlying commodity, parallel salam contracts may be used instead.
Despite the incongruence of Islamic juristsÊ opinions on the permissibility of
parallel salam, there are two venues for parallel salam cited in the existing
literature namely:

(a) Parallel salam with the original seller; and

(b) Parallel salam with a third party.

The first salam involves an arrangement whereby the IFI purchases the
underlying commodity by means of salam to be delivered at the maturity date
and hence enters into a parallel salam with the original seller to sell the
underlying commodity at the same maturity date. The bankÊs selling price would
be higher than the original purchase price and considered justifiable since there
has been a time lapse. However, entering into a parallel salam with the original
seller is argued to be equivalent to a sales and buy back transaction which is
widely criticised in the Islamic commercial jurisprudence.

Whereas, the second salam involves an arrangement whereby the IFI is obliged
to sell the underlying commodity to a third party after taking delivery from the
original seller at the maturity date. The rights and obligations of one salam
contract must be independent of the other. In other words, the IFI is still liable to
deliver the underlying asset to the third party even though the original seller did
not deliver the underlying asset at the maturity date.

10.2.2 Bay’ al-Istisna’ and Ju’alah


Similar to bayÊ al-salam, an istisnaÊ contract allows for deferred delivery of an
underlying asset at a stipulated future date. In contrast, the payment for istisnaÊ
assets can be made in a deferred basis either in lump sum or staggered amount,
where the selling price is agreed and fixed upon.

Whilst bayÊ al-salam is commonly utilised in the market for various sorts of
commodity and agricultural products, bayÊ al-istisnaÊ is applicable for
manufactured goods such as residential homes, factories, aircrafts and
machinery. In a classical istisnaÊ contract, the istisnaÊ buyer or mustani will
TOPIC 10 DERIVATIVE INSTRUMENTS AND ISLAMIC FINANCE  141

request the manufacturer or sani to manufacture the assets according to his


specifications to be delivered upon completion for a consideration of the sale
price to be paid in a lump sum or deferred payment.

However, in the current practice of Islamic banking and finance, as the IFIs are
not willing to take the risks of non-completion and abandoned projects, a parallel
istisnaÊ is rather used to provide financing to their customers which involves the
following:

(a) IstisnaÊ sale contract between customer and bank; and


(b) IstisnaÊ purchase between bank and manufacturer.

On the other hand, juÊalah is a contract of service whereby commission is paid to


the worker as a compensation for the task or duty performed.

10.2.3 Istijrar Contract


What does istijrar mean?

The istijrar is a financing contract where it includes embedded options.

In this contract, the parties have the right to choose or exercise the option to settle
the settlement price at the predetermined murabahah (cost-plus) price if the spot
price of the underlying asset exceeds predetermined bounds such as lower and
upper boundaries.

Similar to other contracts, the istijrar also involves two parties; for example, a
purchaser or buyer which could be a company looking for Islamic financing
instrument to purchase or buy the underlying asset and an Islamic financial
institution (IFI).

Originally, the istijrar contract was introduced as an Islamic financing instrument


in Pakistan. The following illustrations will demonstrate the operational
mechanisms of the istijrar contract.

Illustration 10.1:
Let us say, a firm looking for short-term working capital to finance the
acquisition of a commodity which is needed as raw material approaches an
Islamic bank.
142  TOPIC 10 DERIVATIVE INSTRUMENTS AND ISLAMIC FINANCE

The Islamic bank buys the commodity at the current price (P0), and resells the
commodity to the company for payment to be made at a mutually agreed upon
date in the future, for instance, in six monthsÊ time.

The settlement price that will arise on maturity is contingent on the underlying
assetÊs price movement from t0 to t180; where t0 is the day the contract was
commenced and t180 is the 180th day which would be the maturity day. It is
different from the murabahah contract where the settlement price would simply
be a predetermined price, P*, where P* = P0 (1 + r) and r is just simply profit of
the Islamic bank.

In the case of istijrar, the price settled on the maturity date could either be
murabahah price (P*) or an average price ( P ) of the commodity between the
period t0 and t180.

Therefore, the settlement price in the istijrar contact will be based on how the
prices have behaved and which party chooses to ‰fix‰ the settlement price. Put
differently, one of the parties will choose the settlement price on the maturity
date. This brings us to the concept of options. So the embedded option is the
right to settle the final price on maturity which will happen at any time before
the contract expiration.

At the beginning of the contract, both parties such as the company and the
Islamic bank agree on the following two conditions:
(a) The predetermined murabahah price, P*; and
(b) Upper and lower boundaries around the P0 (the Islamic bankÊs purchase
price at t0).

These conditions can be simplified into Figure 10.2.

Figure 10.2: Graphical representation for Illustration 10.1

Where:
P0 = The price that bank buys the underlying commodity
P* = The murabahah (cost-plus) price P* = P0*(1 + r)
PLB = The price of lower bounds
PUB = The price of upper bounds
TOPIC 10 DERIVATIVE INSTRUMENTS AND ISLAMIC FINANCE  143

Take note that there will be an additional price which is the average price of the
commodity within the boundaries which we denoted as P . Therefore, the
settlement price (PS) is dependent on the movement of the price (see Table 10.2):

Table 10.2: Two Factors of Settlement Price (PS)

PS = P If the prices of underlying asset lie within the boundaries

PS = P* If the price of underlying asset exceeds one of the boundaries and one of
parties has the option to choose the murabahah price which is P*

Bear in mind that either party can exercise or choose its option. Then, they can fix
the settlement price at the murabahah price (P*) if the spot price exceeds the
boundaries at any time during the term of the contract. As to which party would
exercise or choose, this option would be selected according to the direction of the
spot price movement.

Then, the next question will be who will choose the murabahah price (P*)? The
answer depends on how the price behaves during the contract. You can refer to
Table 10.3 for the summary of the settlement price of the istijrar contract.

Table 10.3: Settlement Price of the Istijrar Contract

Price Movement Executions/Strategy


If Pt is lower than the lower bound (Pt < PLB), the The bank exercises:
1 banks loses and the customer gains until
exercise. PS = P*

PS If the price is within the boundaries, PLB < Pt <


2 PS = P
PUB.
If Pt is higher than the upper bounds (PUP > Pt), The customer exercises:
3 the customer loses and the bank gains until
exercise. PS = P*

Where:
PS = The settlement price at maturity
P* = The murabahah (cost-plus) price P* = P0*(1 + r), predetermined
Pt = The spot price underlying asset at maturity t
P = Average price within boundaries
144  TOPIC 10 DERIVATIVE INSTRUMENTS AND ISLAMIC FINANCE

If we analyse the istijrar contract from an optionÊs perspective, it is a complex


contract due to its two different exercise styles combined into one. We can think
of the embedded options in the istijrar as:

The fact that the purchaser, in this case the customer, has the right to
fix the purchasing price at P* when the price goes higher than the
upper boundary which implies that he has a call option at an exercise
price of murabahah price (P*) while the bank a put option at the same
exercise price.

Therefore, unlike the conventional options, the maximum potential gain or loss is
limited. Such a financing contract prevents a fixed return on a riskless asset
which would be considered riba and also there is no room for gharar in that both
parties know up front the murabahah price (P*) and the range of other possible
prices.

10.2.4 Bai-al-Urbun
What does bai-al-urbun mean?

In an urbun contract, the buyer of a product may place with the seller a small
deposit in exchange for which, the seller might grant a period of time, at the
end of which the buyer forfeits his deposit.

If the buyer goes ahead with the transaction within the stipulated time, then the
buyer pays the agreed price less the urbun payment made earlier. The typical
period of time granted in an urbun is three days, similar to a three days call
option.

In the case of options, regardless of whether it is exercised or not, the premium


paid is never recovered while in the case of the urbun, the deposit placed is
deducted from the agreed price to take delivery from the seller.
TOPIC 10 DERIVATIVE INSTRUMENTS AND ISLAMIC FINANCE  145

There are two forms of bai-al-urbun as described in Table 10.4.

Table 10.4: Two Forms of Bai-al-urbun

A Non-refundable Urbun A Refundable Urbun


 The deposit placed is lost to the seller  The deposit placed by the buyer is
if the buyer decides not to go ahead returned to the buyer if he chooses
with the transactions. not to proceed.
 This is also the variant which is more  Acceptable to all four madzhabs.
controversial, with the exception of
the Hanbali madzhab, which permits
it.
 The Maliki, Shafie and Hanafi
madzhabs find it objectionable.

SELF-CHECK 10.2

What are the several Islamic contracts which have similar features of
financial derivatives? Describe them with one example each.

10.3 PRICING OF A SUKUK IJARA WITH


WARRANTS/EMBEDDED OPTIONS
Did you know several ‰exotic‰ sukuk cases that have embedded options were
issued by several companies? An example is The Khazanah Exchangeable Sukuk
and KFC Holdings Sukuk in 1997.

What happens if a warrant is attached to a sukuk? If a warrant is attached to a


sukuk, the value of the sukuk increases due to the amount of the value of the
warrant. Then, what is the warrant? It is nothing else but a long-dated call
option.

How do we determine the value of such a sukuk? In order to determine the value
of such sukuks, we have to determine the value of the call and add the callÊs
value to that of the basic sukuk.

Similar to the conventional call options, we can use the Black-Scholes option
pricing model (BSOPM) to determine the value of the call. As stated before, the
warrant is a call option since it provides a right, but not the obligation, to buy the
stock at a predetermined price. As we know, the BSOPM provides a closed-form
146  TOPIC 10 DERIVATIVE INSTRUMENTS AND ISLAMIC FINANCE

solution to value European style calls (or puts). In the case of the European style
option, one can only be exercised at maturity and not before the maturity.

In order to apply the BSOPM on the valuation of the call, we need to have the
following five parameters:

(a) The current stock price = S0

(b) Estimated volatility of the underlying asset = 

(c) Time to maturity of the call (warrant) as a percentage of a year = t

(d) The risk-free interest rate = rf

(e) The exercise price of the call = k

Then, the BSOPM can be written as:

C = S0.N(d1) ă Ke-rt  N(d2)

Take note that N(d1) and N(d2) are probability values derived from a cumulative
density function and we calculate them as (see Table 10.5):

Table 10.5: Formula for d1 and d2

d1 d2

     d2  d 1   T
2
S  T
ln   r   
K    2 
d1 
 T

In using the BSOPM to value the warrant attached to a sukuk, we do the


following two modifications:

(a) We replace the risk-free interest rate with the three months KLIRR (Kuala
Lumpur Interbank Rate of Return); and

(b) If exercise of the warrant will result in new shares being issued, then an
adjustment has to be made for the dilution factor.

Finally, we will derive the value of the call per share by application of the
BSOPM equation in Table 10.5. Since the warrants generally can be exercised into
more than one share, an additional adjustment needs to be made.
TOPIC 10 DERIVATIVE INSTRUMENTS AND ISLAMIC FINANCE  147

For instance, if the warrant can be exchange into 500 shares, then the call value
derived at by applying the BSOPM should be multiplied by 500 to arrive at the
value of the warrant. Let us say, we get a call value of RM0.50 for per share by
using the BSOPM and the warrant has a conversion ratio of say, 500 shares, then
the value of the embedded option represented by the warrant will be worth
RM250.00 (RM0.50  500).

Since the existence of the embedded options provides that the sukuk holder
potentially profits from the appreciation of the underlying stocks, and since the
ringgit value of that profit potentially has been valued at RM250 by applying the
BSOPM, it is required to add the RM250 to present value (PV) of the other cash
flows determined from the sukuk. If the sukuk al-ijarah has periodic ijarah
payments, then the total value of the sukuk will be:

Present value (PV) of periodic ijarah payments + RM 250 (value of the warrant)

SELF-CHECK 10.3

Demonstrate the pricing process of the sukuk ijara with warrants or


embedded options.

10.4 CURRENT DERIVATIVE INSTRUMENTS


FROM ISLAMIC PERSPECTIVE
There are different views on the validity and permissibility of derivatives from
the Islamic perspective by sharia scholars or jurists. Even where sharia scholars
have found the derivatives are objectionable, their reason for objection also
differs.

So far there is no consensus among the sharia scholars. Most of the works by
sharia scholars have been of a highly juridical nature. They study derivatives
within the contexts of the contractual arrangements and thereby miss the bigger
picture of why they are needed in modern business environments.
148  TOPIC 10 DERIVATIVE INSTRUMENTS AND ISLAMIC FINANCE

10.4.1 Opinions on Futures Contracts


The following are some opinions on futures contracts:

(a) Fatwa of Omam al-Haramaini al-Jauwaini


„The trading of futures is halal if the practice is based on darurah and the
needs or hajaat of the ummah.‰

(b) Sharia Advisory Council (SAC) of Securities Commission, Malaysia

(i) „Futures trading of commodities are approved as long as underlying


asset is halal.

(ii) Crude palm oil futures contracts are approved for trading.

(iii) For stock index futures (SIF) contract, the concept is approved.‰

But the current FBM KLCI based stock index futures (SIF) have non-halal
stocks; it is not approved. Thus, it implies that a SIF contract of a halal
index would be acceptable.

(c) Ustaz Ahmad Allam (Islamic Fiqh Academy ă Jeddah, 1992)


„Stock index futures (SIF) trading is haram, since some of the underlying
stocks are not halal.‰ Until and unless the underlying asset or basket of
securities in the SIF is all halal, SIF trading is not approved.

(d) Mufti Taqi Usmani (Fiqh Academy ă Jeddah)


Futures transactions are not permissible for two reasons:

(i) According to the sharia, sales or purchases cannot be affected for a


future date; and

(ii) In most futures transactions, delivery or possession is not intended.


TOPIC 10 DERIVATIVE INSTRUMENTS AND ISLAMIC FINANCE  149

10.4.2 Opinions on Option Contracts


There are six opinions on option contracts which are:

(a) Ahmad Muhayyuddin Hassan (1986)


Objects to option trading for two reasons:

(i) „Maturity beyond three days as in al-khiyarat is not acceptable.

(ii) The buyer gets more benefits than the seller ă injustice.‰

(b) Abu Sulayman (Fiqh Academy ă Jeddah, 1992)


„It is acceptable when viewed in the light of bai-al-urbun but considers
options to have been detached and independent of the underlying asset ă
therefore, unacceptable.‰

(c) Mufti Taqi Usmani (Fiqh Academy ă Jeddah)


„Promises as part of a contract is acceptable in sharia, however the trading
and charging of a premium for the promise is not acceptable.‰ Yet others
have argued against options by invoking ‰maisir‰ or unearned gains; that
is, the profits from options may be unearned.

(d) El Gari (1993) and Yusuf Qaradawi


El Gari argues, „In favour of call options based on the framework of bai-al-
urbun.‰

Yusuf Qaradawi argues that, „The ruling by Ibn Hanbal on urbun should
be adapted to modern times implying that the use of options could be
justified on the basis of urbun.‰

(e) Hashim Kamali (International Islamic University Malaysia, 1998)


He finds options are acceptable. Invokes the Hanbali tradition, cites
Hadiths of Barira (RA) and Habban Ibn Munqidh (RA). Also draws
parallels with the bai-al-urbun in arguing that premiums are acceptable. He
also cites that contemporary scholars such as Yusuf al-Qaradawi and
Mustafa al-Zarqa have authenticated bai-al-urbun.

(f) Sharia Advisory Council (SAC) Securities Commission, Malaysia


No formal opinion on options. The fact that there are no equity options,
only index options available currently has meant that there is no urgency.
Index options are disallowed based on the argument that some of the stocks
in the FBM KLCI are non-halal. However, the SAC has approved as halal
the trading of warrants or transferable subscription rights (TSRs) as long as
the underlying stock is designated as a halal stock.
150  TOPIC 10 DERIVATIVE INSTRUMENTS AND ISLAMIC FINANCE

SELF-CHECK 10.4
What are sharia opinions on permissibility of financial derivatives?

 The prerequisites of Islamic financing instruments which have to be


eliminated from Islamic transactions are riba (usury), rishwah (corruption),
maysir (gambling), gharar (unnecessary risk) and jahl (ignorance).

 There are other several conditions to be considered in Islamic financial


instruments such as halal and permissible from Islamic perspective.

 Several Islamic financial instruments which have similarities with financial


derivatives are bayÊ al-salam, bayÊ al-istisnaÊ and juÊalah, istijrar contract and
baiÊ al-urbun.

 BayÊ al-salam or forward sales is an Islamic transaction whereby the seller


agrees to undertake the delivery of an underlying asset at the stipulated
future date in exchange for a consideration paid either in cash or in kind by
the purchaser immediately at a spot price.

 An istisnaÊ contract allows for deferred delivery of an underlying asset at a


stipulated future date. It is applicable for manufactured goods such as
residential homes, factories, aircraft and machinery.

 JuÊalah is a contract of service whereby commission is paid to the worker as a


compensation for the task or duty performed.

 The istijrar is a financing contract which includes embedded options; the


parties have the right to choose or exercise the option to settle the settlement
price at the predetermined murabahah (cost-plus) price if the spot price of the
underlying asset exceeds predetermined bounds such as lower and upper
boundaries.

 In an urbun contract, the buyer of a product may place with the seller a small
deposit in exchange for which, the seller might grant a period of time, at the
end of which the buyer forfeits his deposit.
TOPIC 10 DERIVATIVE INSTRUMENTS AND ISLAMIC FINANCE  151

 In order to determine the value of a sukuk, we have to determine the value of


the call and add the callÊs value to that of the basic sukuk. Similar to the
conventional call options, we can use the Black-Scholes option pricing model
(BSOPM) to determine the value of the call.

 There are different views on the validity and permissibility of derivatives


from Islamic perspective by sharia scholars or jurists. Even where sharia
scholars have found the derivatives are objectionable, their reason for
objection also differs. So far there is no consensus among the sharia scholars.

Al-ijarah Bay al-istisna


Bay al-urbun JuÊalah
Bay al-salam Sharia opinions on derivatives

Embedded options Sharia-compliant derivatives

Istijrar Warrants

Bacha, O. I. (2012). Financial derivatives: Markets and applications in Malaysia


(3rd ed.). Shah Alam, Malaysia: McGraw-Hill.

Bacha, O. I., & Mirakhor, A. (2013). Islamic capital markets: A comparative


approach. Singapore: John Wiley & Sons.
MODULE FEEDBACK
MAKLUM BALAS MODUL

If you have any comment or feedback, you are welcome to:

1. E-mail your comment or feedback to modulefeedback@oum.edu.my

OR

2. Fill in the Print Module online evaluation form available on myINSPIRE.

Thank you.

Centre for Instructional Design and Technology


(Pusat Reka Bentuk Pengajaran dan Teknologi )
Tel No.: 03-27732578
Fax No.: 03-26978702

View publication stats

You might also like